You are on page 1of 44

I

NTERNATI
ONALENVI
RONMENTALREGULATI
ONS
ANDNATURALRESOURCESDI
GESTS

MERI
DAV.PEOPLE
G.R.No.158182      
June12, 2008
Ponente:Carpi
o

FACTS:Pet it
ionerwaschar gedint heRTCofRombl onwi thv iol
ationofSect ion68ofPD705f or
"cutti
ng,gat hering,col l
ecti
ngandr emovingal onenar rat r
eei nsideapr i
vatel andov erwhi ch
privatecompl ai
nantOscarTansi ongcocl ai
msowner ship.Whenconf ronteddur i
ngt hemeet i
ng
aboutt hef el
lednar rat r
ee,petit
ioneradmi tt
edcut ti
ngt het r
eebutcl aimedt hathedi dsowi ththe
permi ssionofoneVi carCal i
xwho,accor dingt opet it
ioner,boughtt heMay odPr oper t
yf r
om
Tansiongcoi nOct ober1987underapact oder etrosale.Itwasl aterfoundoutt hatheconv er
ted
thenar ratr
unki ntol umber .
Hewasf oundgui lt
ybyt heTr i
alCour tbutheappeal edt ot heCour tofAppeal sr eiterati
ng
hisdef enseofdeni al.Peti
ti
oneralsocont endedt hatthet ri
alcour tdidnotacqui rej urisdi cti
onov er
thecasebecausei twasbasedonacompl ai
ntf il
edbyTansi ongcoandnotbyaf or estof ficeras
prov i
dedunderSect ion80ofPD705.CAaf fir
medt helowercour t
’srul
ing,butor der edt hesei zed
l
umberconf iscatedi nthegov ernment '
sf avor.Al so,itsust ainedt het r
ialcour t '
sf indingt hat
petiti
onerisboundbyhi sextraj
udicialadmissionsofcut tingt henar ratreeint heMay odPr opert
y
withoutanyDENRper mit
.

I
SSUE:
1) W/Nt hetri
alcour
tacqui redj
uri
sdicti
onoverCri
minalCaseNo.2207eventhoughi
twas
basedonacompl aintfi
l
edbyTansi ongcoandnotbyaDENRf or
estof
fi
cer.–YES.
2) W/Npetit
ioneri
sliableforvi
olat
ionofSecti
on68ofPD705.–YES.

RATI O:
1.TheRev isedRul esofCr i
mi nalPr ocedurel istthecaseswhi chmustbei niti
atedbyacompl aint
fi
ledbyspeci fi
edindi vidual s,non- compl i
anceofwhi choustst hetrialcourtofjuri
sdicti
onf rom
tryi
ngsuchcases.Howev er,t hesecasesconcer nonl ydefamat i
onandot hercr i
mesagai nst
chastityandnott ocasesconcer ningSection68ofPD705.Fur ther,Secti
on80ofPD705doesnot
prohibitani nterest
edper sonf r
om f i
li
ngacompl ai
ntbef or
eanyqual i
fi
edof f
icerforviol
ationof
Sect i
on68ofPD705, asamended.
Mor eover,her e,i twas not" forestof ficers orempl oyees oft he Bureau ofFor est
Dev el
opment ”whor epor tedt oHer nandezt het r
ee-cutti
ngintheMay odPr opertybutTansiongco, a
priv
at ecit
izenwhocl aimsowner shipovert heMay odPr opert
y.Thus, Hernandezcannotbef ault ed
fornotconduct i
ngani nv estigationtodet er mine" i
fthereispr imaf acieevidencetosuppor tt he
compl aintorr eport
."Atanyr ate,Tansiongcowasnotpr ecl
uded, eit
herunderSect i
on80ofPD705
ort heRev i
sedRul es,f rom f il
ingacompl ai
ntbef oret hePr ov i
nci
alPr osecutorforpet i
ti
oner '
s
all
egedv i
olati
onofSect ion68ofPD705.

2)Petiti
onerisguil
toft hesecondpar agraphofsecti
on80,whi chi sthecutt
ing,gat
hering,
coll
ect
ing,orremovingoftimberfrom al
ienableordi
sposabl
epublicland,orf
rom pri
vateland
wit
houtanyaut hor
it
y .Thecour
talsosaidt hatthel
umberor“processedlog“i
scov er
edbyt he
“f
orestproduct
s”ter
mi nPD705,ast helaw doesnotdi
sti
ngui
shbet weenaraw andprocessed
ti
mber.

ATTY.VI
LLEGAS 1
I
NTERNATI
ONALENVI
RONMENTALREGULATI
ONS
ANDNATURALRESOURCESDI
GESTS

PROVI
DENTTREEFARMSV.BATARI
O
G.R.No.92285
March28, 1994
Ponente:Bell
osi
ll
o

FACTS:Pet it
ionerPTFIi saPhi lippi
necor porationengagedi ni ndust ri
alt reepl anti
ng.I tgr ows
gubast reesini tspl antati
onswhi chitsuppl iestoal ocalmat chmanuf act urersol el
yforpr oduct ion
ofmat ches.I nconsonancewi tht hest atepol i
cyt oencour agequal ifi
edper sonst oengagei n
i
ndust rial t
reepl antation,Sec.36, par.(1) ,oftheRev isedFor estryCodeconf er sonent i
ti
esl i
kePTFI
asetofi ncent i
vesamongwhi chi saqual if
iedbanagai nsti mpor tationofwoodand" wood-
deri
v ated"pr oduct s.
On5Apr il1989,pr iv
ater espondentA.J.I nter nationalCor por ation( AJIC)i mpor ted4
container sofmat chesf r
om Indonesi aand2ormor econt ai
ner sofmat chesf rom Si ngapore.On25
April1989,uponr equestofPTFI ,Secr etaryFact oranoft heDENRi ssuedacer ti
ficati
ont hat" t
her e
areenoughav ail
abl esof t
woodsuppl yi nt hePhi l
ippinesf ort hemat chi ndust r
yatr easonabl e
pri
ce. "I nlightoft his,PTFIf i
ledwi t
ht heRTCofMani laacompl ai ntf orinj unct i
onanddamages
withpr ay erforaTROagai nstrespondent sCommi ssionerofCust omsandAJI Ct oenjointhel atter
fr
om i mpor ting mat chesand " wood- der iv
ated"pr oduct s,and t heCol lect orofCust omsf rom
al
lowi ngandr eleasingt hei mpor tati
ons.Thecasewasr affledt or espondentJudgeDemet ri
oM.
Batario.Lowercour tr ul
edinf avorofr espondent s,statingt hati thad" noj ur isdicti
ont odet ermi ne
whatar elegal orill
egal i
mpor t
ations."

I
SSUE:W/ NTheCommi ssi
onerofCustomsunderSec.1207oftheTari
ffandCustomsCodeand
nottheregul
arcour
t,has"excl
usi
vejuri
sdi
cti
ontodeter
minethelegal
i
tyofani mpor
tat
ion,and
otheri
nci
dent
almatt
ersrel
ati
ngtosuch.–YES.

RATI O:Theenf orcementofthei mpor t


ationbanunderSec.36, par.(l
),oftheRev isedFor est
ryCode
i
swi t
hint heexclusivereal
m oft heBur eauofCust oms,anddi r
ectrecourseofpet i
ti
onertothe
RegionalTr ialCourttocompelt heCommi ssionerofCust omst oenforcethebani sdevoidofany
l
egalbasi s.Anor derofaj udget oi mpound,sei zeorf orf
eitmusti nevit
abl ybebasedonhi s
determinat i
onanddecl arati
onoft hei nv ali
dit
yoft hei mportati
on,hence,anusur pationofthe
prerogativeandanencr oachmentont hej urisdi
cti
onoft heBureauofCust oms.
Also,PTFI'
scorrespondencewi t
ht heBureauofCust omscont esti
ngt hel egali
tyofmatch
i
mpor t
ationsmayal r
eadytaket henat ureofanadmi ni
strat
iveproceedingthependencyofwhi ch
woul dprecludethecourtfrom interf
eri
ngwi thitunderthedoctrineofprimaryjurisdicti
on.

ATTY.VI
LLEGAS 2
I
NTERNATI
ONALENVI
RONMENTALREGULATI
ONS
ANDNATURALRESOURCESDI
GESTS

PEOPLEV.CFI
G.R.No.L-46772
Februar
y13, 1992
Ponente:Medialdea

FACTS:Thepr i
vat er espondent swer echar gedwi ththecr i
meofqual ifi
edt heftofl ogs,def
inedand
punishedunderSect ion68ofPr esident i
alDecr eeNo.705,ot herwiseknownast heRev i
sed
ForestryCodeoft hePhi li
ppines.Thei nformat i
onpr ovidedthatGodof r
edoAr rozalandLuisFl ores,
togetherwi t
h20ot herJohnDoeswhosei denti
tiesar estil
lunknown,t hef ir
st-namedaccused
beingt headmi nistratoroft heInfant aLoggi ngCor porati
on,conspiredandent eredthepr ivat
ely-
ownedl andofoneFel icitaci
onPuj alte, t
it
ledint henameofherdeceasedf ather,Macar i
oPr udente,
andpr oceededt oi llegallycut,gather ,andt ake,therefrom,wi t
houtt heconsentoft hesaidowner
andwi thoutanyaut hor i
tyunderal icenseagr eement ,60logsofdifferentspecies.
OnMar ch23,1977,t henamedaccusedf i
ledamot iont oquasht hei nformati
onon2
grounds,t o wit:( 1)t hatt he facts char ged do notconst it
ute an of f
ense;and,( 2)thatt he
i
nformat i
ondoesnotconf orm subst antiall
yt othepr escri
bedform.Tr ialcourtt husdismissedt he
i
nformat i
onbasedont her espondent ’sgr ounds.

I
SSUE:W/Ntheinf
ormati
oncorr
ect
lyandpr
oper
lychar
gedanof
fenseandWONt
het
ri
alcour
thad
j
uri
sdi
cti
onovert
hecase.–YES.

RATI O:Theel ement soft hecr i


meofqual ifiedt heftofl ogsar e:1)Thatt heaccusedcut ,gat her ed,
collectedorr emov edt imberorot herf orestpr oduct s;2)t hatt het imberorot herf orestpr oduct scut ,
gat hered,col lect edorr emov edbel ongst ot hegov er nmentort oanypr i
vatei ndi vidual ;and3)t hat
thecut ti
ng,gat her i
ng,col l
ect ingorr emov ingwaswi thoutaut hor ityunderal icenseagr eement ,
l
ease,l i
cense,orper mi tgr ant edbyt hest at e.Thef ailureoft hei nf ormat i
ont oal l
eget hatt hel ogs
takenwer eownedbyt hest atei snotf atal.Itshoul dbenot edt hatt hel ogssubj ectoft hecompl aint
wer et akennotf r
om apubl i
cf orestbutf rom apr ivat ewoodl andr egister edi nt henameof
compl ainant'sdeceasedf ather ,Macar i
oPr udent e.Thef actt hatonl yt hest atecangr antal i
cense
agr eement ,l
icenseorl easedoesnotmaket hest atet heownerofal lthel ogsandt i
mberpr oduct s
producedi nt hePhi li
ppi nesi ncl udingt hosepr oducedi npr ivatewoodl ands.Thus, owner shipi snot
anessent ialel ementoft heof fenseasdef i
nedi nSect ion60ofP. D.No.705.
Ast ot hesecondi ssuer aised,t her egul arcour tsst i
llhasj ur isdict
ion.Sec.80ofPD705
cov ers2speci fici nstanceswhenaf or estof f i
cermaycommenceapr osecut i
onf ort hev iolationof
theRev i
sedFor est r
yCodeoft hePhi lippines.Thef i
rstaut hor i
zesaf orestof fi
cerorempl oy eeof
theBur eauofFor estryt oar restwi t
houtawar rant ,
anyper sonwhohascommi ttedori scommi tting,
i
nhi spr esence,anyoft heof f
ensesdescr i
bedi nt hedecr ee.Thesecondcov ersasi tuat i
onwhen
anof fensedescr ibedi nt hedecr eei snotcommi ttedi nt hepr esenceoft hef orestof ficeror
empl oy eeandt hecommi ssioni sbr oughtt ohi sat t
ent i
onbyar epor toracompl aint .Inbot hcases,
howev er,thef orestof fi
cerorempl oy eeshal linv estigat et heof fenderandf ileacompl ai
ntwi tht he
appr opr i
ateof fi
ci alaut hor i
zedbyl awt oconductapr eli
mi nar yinvest i
gat i
onandf ilet henecessar y
i
nf ormat ionsi ncour t.Unf ortunat ely, thei nst antcasedonotf allunderanyoft hesi tuat i
onscov er ed
bySect i
on80ofP. D.705.Theal legedof fensewascommi tt
ednoti nthepr esenceofaf or est
officerandnei t
herwast heal legedcommi ssi onr epor tedt oanyf or estof fi
cer .Theof fensewas
commi tt
edi napr ivatel andandt hecompl aintwasbr oughtbyapr iv ateof fendedpar tyt ot hef i
scal .
Assuch,t heOSG wascor recti ni nsist i
ngt hatP. D.705di dnotr epealSect i
on1687oft he
Admi nistr
at i
veCodegi v i
ngaut horityt ot hef iscalt oconducti nv est igationi ntot hemat terofany
cri
meormi sdemeanorandhav ethenecessar yinfor mat i
onorcompl aintpr epar edormadeagai nst
per sonschar gedwi tht hecommi ssi onoft hecr ime.I nshor t
,Sec.80doesnotgr antexcl usi ve
author itytot hef orestof fi
cer s, butonl yspeci alaut hor i
tyt or einfor cet heexer ciseofsuchbyt hose
uponwhom i tisv estedbygener allaw.

ATTY.VI
LLEGAS 3
I
NTERNATI
ONALENVI
RONMENTALREGULATI
ONS
ANDNATURALRESOURCESDI
GESTS

LAGUAV.CUSI
G.R.No.L-44649
Apri
l15,1988
Ponente:Guit
err
ez,
Jr.

FACTS:Thi sisamandamuscasef i
ledagainstr espondentsforclosingal oggingr oadwi thout
authorit
y.Thepr iv
ater espondentsextendedt hatast heact scompl ainedofbyt hepet i
tioners
aroseoutoft helegit
imat eexer
ciseofrespondentEast coastDevelopmentEnt er
pr i
ses’ r
ightsasa
ti
mberl icensee,mor epar ti
cul
arl
yint heuseofi tsloggingroads,therefore,ther esoluti
onoft his
questioni spr operl
yandl egal
lywi t
hintheBur eauofFor estDev el
opment ,citi
ngasaut horit
y
PresidentialDecree(P.D.)No.705.Thel owercour taff
ir
medt herespondent ’
sdef ense, stat
ingt hat
thepet i
tionersmustf ir
stseekr ecoursewiththeBur eauofFor estDev elopmentt odet ermi net he
l
egalityoft hecl osureoft heloggingroads,bef oreseekingredresswi thther egularcour t
sf or
damages.

I
SSUE:W/Ntheregul
arcour
tscantakecogni
zanceofthedamagescasewi
thoutf
ir
stseeki
ngt
he
det
ermi
nat
ionoftheBur
eauregar
dingt
helegal
it
yofthecl
osur
e.–YES.

RATIO:P. D.No.705uponwhi chther espondentcour tbasedi tsorderdoesnotv estanypoweri n


theBureauofFor estDev el
opmentt odet er
mi newhet herornott heclosur eofal oggingr oadis
l
egalori l
legalandt omakesuchdet erminati
onapr e-r
equisi
tebef oreanact i
onf ordamagesmay
bemai ntained.Mor eover,thecompl ai
ntinsti
tutedbyt hepetit
ioner sisclearlyfordamagesbased
ont heallegedi llegalclosureoft heloggingr oad.Whet herornotsuchcl osurewasi llegali
sa
mat t
ertobeest abl i
shedont hepar tofthepet it
ionersandamat tert obedi sprovedbyt hepriv
ate
respondent s.Thisshoul dappr opr
iatel
ybet hr eshedouti naj udicialproceeding.I tisbey ondthe
powerandaut horityoftheBur eauofFor estDev el
opmentt odeter minet heunl awfulclosureofa
passageway ,muchl essawar dordenyt hepay mentofdamagesbasedonsuchcl osure.Notev er
y
acti
vit
yinsideaf orestareai ssubjecttothejurisdicti
onoftheBur eauofFor estDev elopment .

ATTY.VI
LLEGAS 4
I
NTERNATI
ONALENVI
RONMENTALREGULATI
ONS
ANDNATURALRESOURCESDI
GESTS

MUSTANGLUMBERV.CA
G.R.No.104988
June18, 1996
Ponente:Davi
de,
Jr.

FACTS:Aconsol idationoft hreecases.Pet it


ioneri sadomest i
ccor porati
onengagedi nal umber
dealerr egisteredwi tht heBur eauofFor estDev elopment.Respondent sar eDENRSec.Fact oran
andAt t
y.Robl esoft heSpeci al Acti
onsandI nv esti
gati
onsDivisi
on( SAID)oft heDENR.
Act i
ngbasedonani nf or
mat i
on,theSAI Dt eam wenttot hel umber yar
dofpet iti
onerand
basedonasear chwar rant
,wer eablet oexecut eanadmi nist
rativeseizureofdi ff
erentkindsof
l
umber ,t o whi ch t he pet i
tionerf ail
ed to pr oduce upon demand t he document s such as
corr
espondi ngcer ti
ficateofl umberor igi
nandauxi l
iar
yinvoi
ceswhi chshal lprovethelegiti
macyof
thei
rsour ceandor i
gi n.Roblest hensubmi t
tedamemor andum r eporttoFact or
an,or deri
ngt he
cancel l
ati
onofpet i
ti
oner ’sDeal er
sPer mit,
fil
ingofcr i
minalcharges,andconf i
scat
ionoft hetrucks
andl umber s.Lowercour truledi nfavorofrespondent s,st
ati
ngt hatpossessi onoflumberwi thout
permi toraut horit
yi snotacr i
me.

I
SSUE:W/Nalumbercannotbeconsi
deredti
mberandt
hatpet
it
ionershoul
dnotbehel
dfori
l
legal
l
oggi
ngunderSec.68oft
heRevi
sedForestr
yCode.–NO.

RATIO:Whil
ePD705expl ici
tlyprov i
desthattimberisi
ncludedi nthet erm“ for
estpr oducts”,t
he
termlumberisf oundinparagraph( aa)ofSection3whi
chst at
est hatthel at
teri
sapr ocessedlog
orprocessedforestrawmat eri
al.Clearl
y,thelawusest
hewor d“lumber ”init
splai nandcommon
usage,andint heabsenceofal egisl
ati
veintentt
othecont r
ary,itshal lbeint
er pret
edassuch.
Hence,itissaf etoconcludet hatt helaw makesnodi sti
nctionwhet herthef orestproductis
processedornot .Theref
ore,JudgeTer esit
aCapulongcommi tt
edgr av eabuseofdi screti
onin
dismissi
ngthecase.

ATTY.VI
LLEGAS 5
I
NTERNATI
ONALENVI
RONMENTALREGULATI
ONS
ANDNATURALRESOURCESDI
GESTS

TI
GOYV.CA
G.R.No.144640      
June26, 2006
Ponente:Azcuna

FACTS:OnAugust3,1993,Nest orOng,whohadbeenengagedi nt het ruckingbusi nessinIl


igan
Citysince1986,wasal legedlyintr
oducedbyhi sfriendGamadMunt odt oLol ongBer todazowho
signif
iedhisi ntenttor entt hetrucksofOngt otransportconstructionmat erialsfrom Lanaodel
Nor t
et oDi pologCi ty.ACont r
actt oTr ansportwassupposedl yent eredi ntobet weenOngand
Bertodazo.Pet i
tionerTigoyandacer tainSumagangwer et henempl oy edast ruckdr i
v ersofOng.
Duet oadi spatch,poli
cemenappr ehendedt hetrucksdriv
enbyTi goy ,basedonasuspi cion
that“hotitems”wer eloadedt herein.Uponi nspect
ion,thepol i
ceofficersdi scov eredpi l
esofsawn
l
umberbeneat ht hecementbagsi nbot htrucks,anduponi nquir
y,thedr i
v erscoul dnotpr oduce
anyper mitforthel umber .ThereaftertheForesteroftheDENRnamedDi ngali nv esti
gatedandt hen
chargedpet it
ionerwi thv i
olati
onofSect i
on68ofPD705f ori
ll
egalpossessi onofl umberwi t
hout
permit.

I
SSUE:W/
NTi
goyi
sgui
l
tyofconspi
racyi
npossessi
ngort
ranspor
ti
ngl
umberwi
thoutper
mit
.–
YES.

RATI O:Ther ear et woway sofv iolatingSect i


on68oft heabov eCode:1)bycut ting,gat her
ing
and/orcol l
ect i
ngt i
mberorot herf orestpr oduct swi t
houtal icense; and,2)bypossessi ngt i
mberor
otherf orestpr oduct swi thoutt her equired legaldocument s.Petiti
onerwaschar ged wi thand
conv i
ctedoft ranspor ti
ngl umberwi thoutaper mitwhi chi spuni shableunderSect ion68oft he
Code.He, Sumagangandt her estoft heircompani onswer eappr ehendedbyt hepoliceof fi
cersin
fl
agrant edelict
oast heywer etranspor ti
ngt hesubj ectlumberf rom Lar apant oDipologCi ty.
Petit
ionermai ntainst hathecoul dnothav econspi redwi thLol ongBer todazoashedi dnot
knowaboutt heunl icensedl umberi nt het r
ucks.Hebel ievedt hatwhathewast ranspor t
ingwer e
bagsofcementi nv i
ewoft hecont ractbet weenOngandBer todazo.Al so, hewasnotar oundwhen
Bertodazol oadedt het r
uckswi tht hel umberhi ddenundert hebagsofcement ,howev er,thecourt
i
sunconv i
ncedbecauseTi goyr efusedt ost opatacheckpoi ntandwhen accost ed,evenof f
ered
greasemoneyt ot hepol icemen.I nof fensesconsi deredasmal apr ohi
bita,thecommi ssionoft he
prohibitedacti sthecr i
mei t
self.I tissuf f i
cientthatt heoffenderhast hei ntenttoper petratethe
actpr ohibi
tedbyt hespeci al law, andt hati tisdoneknowi nglyandconsci ously.

ATTY.VI
LLEGAS 6
I
NTERNATI
ONALENVI
RONMENTALREGULATI
ONS
ANDNATURALRESOURCESDI
GESTS

PEOPLEV.QUE
G.R.No.120365
December17,1996
Ponente:Puno

FACTS: Accused-appell
antWi lsonQueappeal sfrom hisconv ict
ionforv i
olat
ionofSect i
on68ofPD
705.
Thef act
sshow t hatt woweeksbef oreMar ch8,1994,amemberoft heProv i
ncialTask
ForceonI ll
egalLogging,receivedaninf ormationt hatat en-
wheelertruckloadedwi t
hi l
legallycut
l
umberwi l
lpassthroughIlocosNor te.Act ingonsai di nformati
on,member softhePr ovi
ncialTask
Forcewentonpat r
olseveraltimeswi t
hint hev i
cinit
yofGener alSegundoAv enuei nLaoagCi t
yand
eventuall
ysawt hetruck.Ther ewer ethreeper sonsonboar dthetruck:dr i
verCacao,Wi l
sonQue,
whowast heownerofsai dt ruck,andanunnamedper son.Thepol i
cet hencheckedthecar goand
foundthatitcontai
nedcoconutsl abs,butinsertedt hereinwher esewnl umber,asadmi ttedbyQue
himself.Whenr equir
edt oshowaper mit ,
Quef ai
ledt odosoandt huswaschar gedforviolati
onof
Sec.68ofPD705.

ISSUE:W/ Npeti
ti
onerv
iol
atedSect i
on68ofP.D.705becauseE.O.277t hatamendedSecti
on68,
which penali
zesthepossession ofti
mberorot herforestproduct
swi thouttheproperlegal
document s,
didnoti
ndi
cat
et heparti
cul
ardocumentsnecessar
ytomaket hepossessi
onlegal
,and
consider
ingthatot
herl
awsandr egulat
ionsdi
dnotexistattheti
meoft heenactmentofsai
dE. O.
–YES.

Held:Appel lantinterpr etst hephr ase“ exi stingfor estlawsandr egul ati
ons”t or ef ertot hosel aws
andr egulationswhi chwer eal readyi nef fectatt het i
meoft heenact mentofE.O.277. The
suggest edi nterpretationi sstrainedandwoul dr endert hel aw i nut i
le. St atutoryconst ruction
shoul dnotki llbutgi v el i
fet ot hel aw. Thephr aseshoul dbeconst ruedt or efert ol awsand
regul at
ions exi sti
ng att he ti
me ofpossessi on oft imberorot herf orestpr oduct s. DENR
Admi ni
strativeOr derNo.59ser iesof1993speci fiest hedocument sr equi redf ort het r
anspor tof
ti
mberandot herf or estpr oduct s.ThusQue’ spossessi onoft hesubj ectl umberwi thoutany
document at i
oncl earlyconst it
utesanof fenseunderSect i
on68ofP. D.705.
Also,t hecour trejectedQue’ sar gumentt hatt hel aw onl ypenal i
zespossessi onofi l
legal
forestpr oduct sandt hatt hepossessorcannotbehel dl iableifhepr ov esthatt hecut ti
ng, gather i
ng,
collecti
ngorr emov alofsuchf or estpr oduct sisl egal.Ther ear e2di sti
nctandsepar ateof fenses
puni shedunderSect ion68ofP. D.705, t
owi t:
(1)Cut t
ing, gat heri
ng, col l
ectingandr emov i
ngt i
mberorot herf orestpr oduct sf rom
anyf or estl and,ort i
mberf rom al ienabl eordi sposabl epubl icl and,orf rom
pri
vatel andwi thoutanyaut hor i
ty;and
(2)Possessi onoft i
mberorot herf orestpr oduct swi thoutt hel egaldocument s
requir
edunderexi st ingf orestl awsandr egulat i
ons.
Int hef ir
stof fense, onecanr aiseasadef enset hel egali
tyoft heact sofcut ti
ng, gather i
ng,
collecti
ngorr emov i
ngt imberorot herfor estpr oduct sbypr esentingt heaut horizationi ssuedbyt he
DENR.I nt hesecondof fense,howev er,i tisi mmat eri
alwhet hert hecut ting,gat her i
ng,col lecting
andr emov aloft hef or estpr oduct sisl egalornot .Mer epossessi onoff or estpr oduct swi thoutt he
properdocument sconsummat est hecr ime.Whet herornott hel umbercomesf rom al egalsour ce
i
si mmat erialbecauseE. O.277consi derst hemer epossessi onoft imberorot herf orestpr oduct s
withoutt hepr operl egal document sasmal um pr ohibit
um.

ATTY.VI
LLEGAS 7
I
NTERNATI
ONALENVI
RONMENTALREGULATI
ONS
ANDNATURALRESOURCESDI
GESTS

CALUBV.CA
G.R.No.115634
Apri
l27,2000
Ponente:Qui
sumbi
ng

FACTS:t heFor estPr otect i


onandLaw Enf orcementTeam oft heCommuni tyEnv i
ronmentand
NaturalResour cesOf fice( CENRO)oft heDENRappr ehended2mot orvehi cleswher einConst ancio
AbugandaandPi oGabon,t hedr i
ver soft hev ehicles,f ailedt opr esentpr operdocument sand/ or
l
icenses.Thus,t heappr ehendi ngt eam sei zedandi mpoundedt hev ehiclesandi tsloadofl umber .
Petit
ioner ,Feli
peCal ub, Pr ov incialEnv ironmentandNat uralResour cesOf fi
cer ,thenf iledacr iminal
compl aintagai nstAbuganda, f
orv i
olat ionofSect ion68ofPD705asamendedbyExecut i
veOr der
277,(Rev isedFor estryCode) .Lowercour truledi nf av orofaccused,andev engr ant edr ecover yof
possessi ont ot hem v iar epl ev i
n.
Uponpet i
tioner ’
sappeal ,theCour tofAppeal sdeni edsai dpet i
tion,st atingt hatt hemer e
seizureofamot orv ehi clepur suantt ot heaut hor i
tygr ant edbySect ion68ofP. D.No.705as
amended byE. O.No.277 does notaut omat icallypl ace sai d conv ey ance i n cust odial egi
s.
Accor dingt ot heappel latecour t,suchaut hor i
tyoft heDepar t
mentHeadoft heDENRorhi sdul y
author i
zedr epresent ativet oor dert heconf i
scat i
onanddi spositionofi l
legallyobt ainedf orest
product sandt heconv ey anceusedf ort hatpur posei snotabsol uteandunqual if
ied.I tissubjectt o
perti
nentl aws, regul ati
ons, orpol i
ciesont hatmat ter,addedt heappel l
atecour t.
TheDENRAdmi ni strativeOr derNo.59, seriesof1990, isonesuchr egul ation, theappel l
ate
courtsai d.Addi t
ionally,r espondentCAnot edt hatt hepet itioner sf ail
edt oobser vet hepr ocedur e
outli
nedi nDENRAdmi nist r
at iveOr derNo.59, seriesof1990.Theywer eunabl et osubmi tar eport
ofthesei zuret ot heDENRSecr etary ,togi veawr i
ttennot icet ot heowneroft hev ehicl
e,andt o
renderar eportoft hei rf indingsandr ecommendat ionst ot heSecr etary .Mor eov er,petiti
oners’
fai
luretocompl ywi tht hepr ocedur el aiddownbyDENRAdmi nistr
ativeOr derNo.59, ser i
esof1990,
wasconf irmedbyt headmi ssi onofpet itioner s’counselt hatnoconf iscationor derhasbeeni ssued
pri
ort ot hesei zur eoft hev ehicleandt hef il
ingoft her epl ev i
nsui t.Ther ef ore,inf ailingtof oll
ow
suchpr ocedur e,accor di ngt ot heappel latecour t,thesubj ectv ehi
cl escoul dnotbeconsi deredi n
custodial egis

I
SSUE:
W/Nsai
dmot
orv
ehi
clesar
eincust
odi
all
egi
spur
suantt
oSect
ion68ofPD705.–YES.

RATI O:Uponappr ehensi onoft hei ll


egally-
cutt i
mberwhi l
ebei ngt ransportedwi thoutper ti
nent
document st hatcoul dev idencet itletoorr i
ghtt opossessi onofsai dtimber,awar r
ant lesssei zure
ofthei nvolvedv ehiclesandt hei rloadwasal lowedunderSect i
on78and89oft heRev i
sedFor est r
y
Code.Not ef urther t hat pet itioners’f ail
uret o obser vet he pr ocedur e out l
ined i n DENR
Admi nistr
ativeOr derNo.59,ser i
esof1990wasj ust if
iablyexpl ained.Pet it
ioner sdidnotsubmi ta
reportoft hesei zur etot heSecr et arynorgiv eawr i
ttennot icet ot heowneroft hev ehiclebecause
ont he3r ddayf ollowingt hesei zur e,GabonandAbuganda,dr i
v ersoft hesei zedv ehicles,forcibly
tookt heimpoundedv ehi clesf rom t hecust odyoft heDENR.Thenagai n,whenoneoft hemot or
vehicleswasappr ehendedandi mpoundedf orthesecondt ime, thepet iti
oners,againwer enotabl e
torepor tthesei zuretot heDENRSecr etar
ynorgi veawr it
tennot icetot heowneroft hev ehicle
becausepr i
v ater espondent si mmedi at
elywentt ocour tandappl i
edf orawr i
tofr epl evi
n.The
seizureoft hev ehicl
esandt heirl oadwasdoneupont heirappr ehensionf orav iolati
onoft he
RevisedFor est ryCode.I twoul dbeabsur dtor equireaconf i
scat ionor derornot i
ceandhear ing
beforesai dsei zurecoul dbeef f
ect edundert heci r
cumst ances.
Hence,si ncet her ewasav i
olati
onoft heRev isedFor estryCodeandt hesei zurewasi n
accor dancewi thl aw,inourv iewt hesubj ectvehicleswer ev al
idlydeemedi ncust odi
al egis.Itcoul d
notbesubj ectt oanact i
onf orr eplevin.Foritispr oper tylawf ullytakenbyv irt
ueofl egalpr ocess
andconsi der edi nthecust odyoft hel aw,andnotot herwi se

ATTY.VI
LLEGAS 8
I
NTERNATI
ONALENVI
RONMENTALREGULATI
ONS
ANDNATURALRESOURCESDI
GESTS

FACTORANV.CA
G.R.No.93540
December13,1999
Ponente:DeLeon,
Jr.

FACTS:OnAugust9,1988,2pol iceof fi
cersoft heMar i
kinai nt
erceptedasi x-wheelert r
uck,
carryi
ngnar r
alumberasi twascr uisingal ongtheMar cosHi ghway .Theyappr ehendedt hetruck
driver
,pr i
v at
er espondentJesus Sy ,and br oughtt he t r
uck and i ts cargo t ot he Per sonnel
InvestigationCommi t
tee/Speci alAct ionsandI nvestigat i
onDi vision( PIC/SAID)oft heDENROf f i
ce
i
nQuezonCi t
y. Ther e,pet it
ionerAt ty.VicenteRobl esoft hePI C/SAIDi nvestigatedt hem,and
discover edt hedi scr epanci esi nt hedocument at
ionoft henar ral umber .Duet ot hef ail
ureof
respondent stoshowt her equireddocument s,peti
tionerFact oran,thenSecr et
ar yofEnv i
ronment
andNat uralResour cesi ssuedanor derfort heconfiscat i
onoft henar ralumberandt hesix-wheeler
truck.Pr iv
at erespondent snei theraskedf orreconsi derati
onofnorappeal ed,thesai dor dertot he
Of f
iceoft hePr esident.Consequent ly,t
hesei t
emswer ethenf orfeit
edi nfavoroft hegov ernment .
Theywer esubsequent lyadv erti
sedt obesol datpubl icauct i
ononMar ch20, 1989.
Respondent sthenf il
edf orpr eliminar yinjuncti
onandr eplevin,t owhi cht het r
ialcour tacceded.
Pet i
ti
onert henr efusedt oobeyt hewr itofseizureandf il
edacount erbond,towhi cht hecour t
deniedbecauseofl ackofser vicetot herespondent s.Cour tofappeal saffi
rmedt helowercour t’
s
decision.

I
SSUE:WONr espondent
scanv
ali
dlyber
est
oredpossessi
onoft
hei
rtr
ucksandl
umberbasedon
t
hewritofr
epl
evin.–NO.

RATI O:Fi rstl


y ,her einr espondent snev erappeal edt heconf iscat i
onor derofpet iti
onerSecr etaryt o
theOf ficeoft hePr esidentaspr ov i
dedf orinSec.8ofP. D.No.705.
Thedoct rineofexhaust ionofadmi nistrati
ver emedi esi sbasi c.Cour t
s, forreasonsofl aw,
comi tyandconv eni ence,shoul dnotent ertainsui tsunl esst heav ail
ableadmi nistrati
v eremedi es
hav efirstbeenr esor tedt oandt hepr operaut hori
tieshav ebeengi venanappr opr i
at eoppor tuni t
y
toactandcor rectt heiral legeder rors,ifany ,commi t
tedi nt headmi nistrati
v efor um.Howev er,
pet i
ti
oner swai v edt hisgr oundf orf ail
uret or aisesuchi nt heirmot iont odi smi ss.Nev ert
heless,i n
orderf orr eplev i
nt opr osper ,thewr ongfuldet ent i
onbyt hedef endantoft hepr oper ti
essoughti nan
actionf orr epl ev inmustbesat isfactoril
yest abli
shed.I fonl yamechani sti
cav ermentt hereofi s
offered, thewr itshoul dnotbei ssued.I nt hecaseatbar ,thesubj ectnar ral umberandsi x-wheel er
truckwer econf iscat edbypet iti
onerSecr etarypur suantt oSect i
on68- A ofP. D.No.705,as
amendedbyExecut i
veOr der( E.
O. )No.277.
Proper tyl awf ull
yt akenbyv irt
ueofl egalpr ocessi sdeemedt obei ncust odial egis.Whena
thingisi nof ficialcust odyofaj udicialorexecut i
veof f
iceri npur suanceofhi sexecut i
onofal egal
wr i
t,replev i
n wi llnotl i
et or ecov erit.Other wi se,ther ewoul dbei nter f
erencewi tht hepossessi on
bef oret hef unct i
onofl aw hadbeenper for medast ot hepr ocessunderwhi cht hepr opert
ywas
taken.Last l
y, Sec.80ofP.D.No.705whi chr equi r
esdel iveryoft hesei zedf orestpr oduct swi thin6
hour sf rom t het i
meoft hesei zuret ot heappr opr i
ateof ficialdesi gnat edbyl aw t oconduct
preliminar yinv est igat ionsappl i
esonl ytocr imi nalpr osecut i
onspr ov i
dedf orinSec.68,andnott o
admi nistrativeconf iscat ionpr ovidedf orinSect i
on68- A.

ATTY.VI
LLEGAS 9
I
NTERNATI
ONALENVI
RONMENTALREGULATI
ONS
ANDNATURALRESOURCESDI
GESTS

PAATV.CA
G.R.No.111107
January10,1997
Ponente:Torr
es,
Jr.

FACTS:OnMay19,1989,t het ruckofpr i


vat
er espondentVi ctoriadeGuzmanwhi l
eoni tswayt o
Bulacanf rom Cagay an,wassei zedbyDENRper sonnelinNuev aVi zcay abecauset hedr i
vercould
notpr oduce t he required document sf ort he f or
estpr oduct sf ound conceal ed int he t
ruck.
PetitionerJov itoLay ugan,t heCommuni tyEnv i
ronmentandNat ur alResour cesOf fi
cer( CENRO)i n
Aritao,Cagay an,issuedanor derofconf i
scationoft hetruckandgav et heowner15day swithin
whicht osubmi tanexpl anationwhyt het ruckshoul dnotbef orf eited. Pr i
v ater espondent s,
howev er,fai
ledt osubmi tther equi redexplanat i
on.Lat er
,theRegi onalExecut iv
eDi rectorofDENR
sust ained pet iti
onerLay ugan’sact ionofconf iscati
onand or der edt he f orfeitureoft hetruck
i
nv oki ng Sect ion68- AofPr esident i
alDecreeNo.705asamendedbyExecut iveOr derNo.277.
Respondent st henappeal ed.
Pendingr esolutionhowev eroft heappeal ,asui tf orr epl ev inwasf iledbyt hepr i
vate
respondent sagai nstpet iti
onerLay uganandExecut i
veDi rector,whi cht hereaf t
eri ssuedawr it
order i
ngt her eturnoft het ruckt opr ivat
erespondent s.Petiti
onerLay uganandExecut i
veDi r
ector
Baggay an fil
ed a mot ion t o di smi ss witht he t r
ialcour tcont endi ng,i nteral i
a,t hatpr i
vate
respondent shadnocauseofact ionf ortheirfailuretoex haustadmi nistrat
iver emedi es.Thet ri
al
cour tdeni edt hemot i
ont odi smi ss, whichtheCAaf f
irmeduponpet itioner ’
sappeal .

I
SSUES:
1. W/N anact i
onf orreplevi
nprospert or ecoveramov abl
epropert
ywhi chi sthesubject
matterofanadmi ni
strati
veforfei
tureproceedingintheDENRpur suanttoSect i
on68-Aof
P.D.705.–NO.
2. W/Nt heSecretaryofDENRandhi srepresentat
ivesempoweredt oconfiscateandfor
f ei
t
conveyancesusedintransport
ingill
egalforestproduct
sinfav
oroft hegov er
nment.–YES.

RATI O: Fi r
stly,theCour thel dt hatbef or eapar tyi sal l
owedt oseekt hei nter vent i
onoft hecour t,iti
s
apr e-condi tiont hatheshoul dhav eav ailedofal lthemeansofadmi ni st
rativepr ocessesaf for ded
him.Thepr emat ureinv ocat i
onofcour t’si nter vent ioni sf ataltoone’ scauseofact i
on.I nt hecase
atbar , therei snoquest iont hatt hecont rov ersywaspendi ngbef oret heSecr etar yofDENRwheni t
wasf or wardedt ohi mf ollowingt hedeni albyt hepet i
ti
oner soft hemot ionf orr econsi derat ionof
privater espondent sthrought heor derofJul y12,1989.I nt heirl etterofr econsi der ationdat ed
June28,1989,pr ivater espondent scl ear lyr ecogni zet hepr esenceofanadmi nistrativef or um t o
whicht heyseekt oav ai
l,ast heydi dav ail
, inther esol utionoft heircase.
Secondl y,ast othepoweroft heDENRt oconf i
scat e,“SECTI ON68- A.Admi nist rati
v eAut hor ityof
theDepar t
mentorHi sDul yAut horizedRepr esent ativeToOr derConf iscat ion.I nal lcasesof
viol
ationoft hisCodeorot herf orestl aws,r ulesandr egul ati
ons,t heDepar tmentHeadorhi sdul y
author izedr epr esentati
v e,mayor dertheconf iscat ionofanyf orestpr oduct si ll
egal lycut ,
gat her ed,
remov ed,orpossessedorabandoned,andal lconv eyancesusedei therbyl and,wat erorai ri nt he
commi ssion oft heof fenseand t o di sposeoft hesamei n accor dancewi th per t
inentl aws,
regulat i
onsandpol i
ciesont hemat ter.”
Itis,t hus,clearf rom t hef oregoi ngpr ov isiont hatt heSecr etaryandhi sdul yaut hor ized
represent ati
vesar egi vent heaut hor i
tyt oconf i
scat eandf orfeitany conv ey ancesut il
izedi n
viol
atingt heCodeorot herf orestl aws, r
ul esandr egul ations.
Last l
y,ast ot hecont entiont hatsi ncet heyar enotl i
ablef orqual i
fiedt hef t,thent hey
shouldnothav enecessar il
yhav ecommi t t
edacr imeunderSec.68.Thi sisunmer it
or i
ous.Wi tht he
i
nt r
oduct ionofExecut iveOr derNo.277amendi ngSect i
on68ofP. D.705,t heactofcut ting,
gather ing,col lecting,remov ing,orpossessi ngf or estpr oductswi thoutaut hor it
yconst itutesa
disti
nctof fensei ndependentnowf rom t hecr imeoft hef tunderAr ti
cles309and310oft heRev ised
PenalCode,butt hepenal tytobei mposedi st hatpr ovidedf orunderAr ticle309and310oft he
Rev i
sedPenalCode.Thi siscl earf rom t hel anguageofEx ecut i
veOr derNo.277wheni telimi nat ed
thephr ase“shal lbegui ltyofqual ifi
edt hef tasdef inedandpuni shedunderAr ticles309and310of
theRev i
sedPenalCode”andi nsertedt hewor ds“shal lbepuni shedwi tht hepenal tiesi mposed

ATTY.VI
LLEGAS 10
I
NTERNATI
ONALENVI
RONMENTALREGULATI
ONS
ANDNATURALRESOURCESDI
GESTS

underAr
ti
cle309and310oft
heRev
isedPenal
Code”
.

ALVAREZV.PI
COP

ATTY.VI
LLEGAS 11
I
NTERNATI
ONALENVI
RONMENTALREGULATI
ONS
ANDNATURALRESOURCESDI
GESTS

G.R.No.162243       
December3,2009
Ponente:Chi
co-
Nazario

FACTS:PI COP( PaperIndustri


esCor pofthePhi l)f i
l
edwi t
ht heDENRanappl icati
ont ohav eits
TimberLicenseAgr eement( TLA)No.43conv ert
edi ntoanI FMA( I
ntegr
atedfor estmanagement
agreement).Inthemi ddleoft heprocessi
ngofPI COP’ sapplicati
on,howev er
,PI COPr efusedt o
att
endf ur
thermeet i
ngswi t
ht heDENR.Instead,on2Sept ember2002, PI
COPf il
edbef or
etheRTC
ofQuezonCi tyaPetiti
onforMandamus1agai nstthenDENRSecr et
aryHehersonT.Al var
ez.PICOP
seekst heissuanceofapr ivil
egedwr i
tofmandamust ocompelt heDENR Secr etar
yt osign,
executeanddel i
veranIFMAt oPI COP.Lowercour truledi
nfav orofrespondents, CAaf f
ir
med, but
uponappealt otheSupr emeCour t,i
trever
sedt hej udgmentoft heCA.NowPI COPf i
l
esaMot ion
forReconsiderat
ion.

ISSUE:W/ NPICOPisenti
tl
edtot heIFMAbywayofmandamusandduet ot
henon-
impai
rment
clauseoft
heConsti
tut
ioni
nrel
ationtoDocument1969.–NO,
Document1969i
snotacont
act
,but
amer ecol
lat
eral
under
taki
ngpursuanttot
heTLA.

RATIO:Anexami nati
onoft hePr esi denti
alWar ranty( Doc.1969)atoncer ev ealsthatitsimply
reassuresPI COP oft hegov ernment ’
scommi tmentt ouphol dthet ermsandcondi ti
onsofi ts
ti
mberl icenseandguar anteesPI COP’ speacef ulandadequat epossessionandenj oymentoft he
areaswhi char ethebasicsour cesofr awmat erialsf oritswoodpr ocessingcompl ex.Thewarranty
coversonl yther i
ghtt
ocut , collect,andr emov et i
mberi nitsconcessionar ea, anddoesnotext end
totheut il
izati
onofot herr esour ces,suchasmi neralr esources,occurr
ingwi t hintheconcession.
ThePr esidenti
alWar r
antycannotbeconsi deredacont ractdisti
nctfr
om PTLANo.47andFMANo.
35.Iti smer elyacol later
alunder taki ngwhi chcannotampl i
fyPICOP’sr ight sunderi t
st i
mber
l
icense.Ther ulinginOposav .Factorant hatat i
mberl icensei snotacont ractwi thi
nt hepurv
iewof
thenon- i
mpai r
mentclausei sedi fy
ing.
Also,PICOP failed t o secur eNCI P Cer if
ication based on R.A.8371 and Sangguni an
consultati
onandappr ov alf orenv ironment al
lycr i
ti
calpr ojectsbasedont heLocalGov er
nment
Code.

ALVAREZV.PI
COP

ATTY.VI
LLEGAS 12
I
NTERNATI
ONALENVI
RONMENTALREGULATI
ONS
ANDNATURALRESOURCESDI
GESTS

G.R.No.162243
November29, 2006
Ponente:Chi
co-Nazar
io

FACTS:Thi si saconsol i
dationoft hr eecases.On23December1999, thenDENRSecr et aryCer il
l
es
promul gat ed DENR Admi nistrati
ve Or der( DAO)No.99- 53 whi ch had f ori ts subj ect,the
"RegulationsGov erningt heI ntegrat edFor estManagementPr ogr am (IFMP)."
Ina28August2000
l
ettert ot heCommuni tyEnv ir
onmentandNat ur alResour cesOf fi
ce( CENRO) ,PICOPsi gnifi
edi ts
i
ntent i
ont oconv er tit
sTLA No.43i ntoanI nt egratedFor estManagementAgr eement( I
FMA)
i
nv okingt hepr ov i
sionsofSect ion9,Chapt erIIIofDAONo.99- 53.Pur suanttotheappl i
cationf or
conv ersi
on, thePer formanceEv aluat ionTeam ofDENRconduct edar eportindicati
ngv iolati
onsby
PICOPofexi sti
ngDENRRul esandRegul ati
onsgov erningTLANo.43, suchast henon- submi ssi
on
ofitsf iv
e- yearf orestpr otect i
onpl anandsev en- yearr eforestationpl anasr equiredbyt heDENR
rul
esandr egul ations,aswel lasov erdueandunpai df or estchar ges.Uponsubmi ssi onoft he
memor andum t ot heDENRSecr etar y,negot i
ationswer ehel dtoassur ePICOP’ scompl i
ancewi th
DAO99- 53, howev erPI COPl atercl aimedt hattheconv ersionhadal r
eadybeencompl et edpur suant
toSec.Al v ar ez’letterofcl earance.
PICOPt henf il
edf orapet itionf ormandamusagai nstpet i
ti
onerwhi chwasgr ant edbyt he
tri
alcour t.Lat eron,Gozunwassubst it
utedi nt hecaseast henew DENRSecr etar y.Cour tof
Appeal sev entual l
yaf fi
rmedt hel owercour t’sruling, t
owhi chher einpet i
ti
onersnowappeal .

I
SSUE:W/
NPICOPisent
it
ledbywayofmandamustotheconver
sionofit
sTLAtoIFMA,andWON
t
her
ewascompl
iancewi
thDAONo.99-53andt
husconver
sionhadalr
eadybeenef
fect
ed.–NO.

RATI O:A t imberl icenseagr eementi snotacont ract ,nei theri st hepr esident ialwar rant yof
Presi dentMar cosacont r
act .Thear gumentt hatt hePr esi dent i
alWar rant yi sacont ractont he
groundt hatt her ewer emut ualconsi der at i
onst akeni nt oaccountconsi stingi ni nv est ment son
PICOP’ spar ti spr epost erous.Al ll i
censeesputupi nv est ment sinpur sui ngt hei rbusi nesses.To
const ruet hesei nv est ment sasconsi derat ioni nacont r actwoul dbet ost ealthilyr enderi neffective
theset tl
edj ur i
spr udencet hat"al icenseoraper mi tisnotacont ractbet weent hesov ereigntyand
thel icenseeorper mi ttee,andi snotapr oper tyi nt heconst i
tut i
onalsense,ast owhi cht he
const itutional pr oscr iptionagai nstt hei mpai r
mentofcont ractsmayext end. "
Ast ot hecompl iancewi tht heDAO,t hef ollowi ngar ether equi sitesf ort heaut omat ic
conv er sionoft heTLAi nt oanI FMA, t
owi t:
1.TheTLA hol derhadsi gni f
iedi tsi nt entt oconv erti t
sTLA i nt oanI FMA pr i
ort ot he
expi rationofi tsTLA;
2.Pr operev aluat ionwasconduct edont heappl i
cat ion; and
3.TheTLAhol derhassat i
sf act ori
lyper formedandcompl i
edwi t
ht het er msandcondi ti
ons
oft heTLAandt heper t
inentr ulesandr egul ati
ons.
Int hecaseatbar ,PICOPf ailedt ocompl ywi t hDAO,asev idencedbyt hememor andum
submi ttedt ot heDENR Secr etar ybyt hePer for manceEv aluationTeam.Fi nal ly,theDENR,by
wit hhol dingt heconv ersi onofPI COP’ sTLANo.43i nt oanI FMA,hasmadeaf act ualf indingt hat
PICOPhasnoty etcompl i
edwi tht her equi rement sf orsuchaconv ersion.Fi ndi ngsoff act sof
admi ni strativeagenci esar egener allyaccor dedgr eatr espect ,ifnotf inality,byt hecour tsbecause
oft hespeci al knowl edgeandexper ti
seov ermat ter sf allingundert hei rjurisdi ction.
Last l
y ,ast owhet herornotconv ersional r
eadyt ookpl ace,t hecour tr uledi nt henegat i
v e.
Bygi v ingt hiscl earancef ort heconv ersionofPI COP’ sTLA i ntoanI FMA,t heDENRSecr etar y
cannot ,byanyst r etchofi magi nat ion, becl aimedt ohav egr antedt heconv er sioni tself.Thel etteri s
cleart hatt he" conv er sion"coul dnotbef i
nalsi ncei tscondi ti
onsanddet ai lsst i
llhav et obe
discussedasst at edi nt hesecondpar agr aphofsai dl et ter ;hence,t hesamel et t
ercoul dnothav e
reducedt oamer ef or mal itytheappr ovaloft heconv er sionofPI COP’ sTLANo.43i ntoanI FMA.
Ev enassumi ng,howev er,t hatt heI FMAhasal readybeenconv erted,t hi si sal lpur el yacademi c
becauseoft heabov e-discussedset t
ledj ur isprudencet hatl oggi ngper mitsar enotcont r
act swi thin
theNon- Impai rmentCl auseandt hus,canbeamended,modi f
ied,r eplacedorr esci ndedwhent he
nat ionali nter estsor equi res.I ft heDENRSecr etary,t her efor e,findst hatt heI FMAwoul dbei n
violat i
onofst at ut es,r ul esandr egul ati
ons,par ticular lyt hosepr otectingt her i
ght soft hel ocal

ATTY.VI
LLEGAS 13
I
NTERNATI
ONALENVI
RONMENTALREGULATI
ONS
ANDNATURALRESOURCESDI
GESTS

government
sand theindi
genouspeopl
eswi thi
nt heIFMA area,then i
tbehoov
estheDENR
Secretar
ytorev
okesuchIFMA.Thesesamest at
utes,r
ulesandregulat
ionsar
ethev
erysame
requi
rement
smenti
onedabovefort
heconver
sionoftheTLANo.43i ntoanIFMA.

DYV.CA
G.
R.No.121587

ATTY.VI
LLEGAS 14
I
NTERNATI
ONALENVI
RONMENTALREGULATI
ONS
ANDNATURALRESOURCESDI
GESTS

March9,1999
Ponent
e:Mendoza

FACTS:OnMay31, 1993, theMay orofBut uanCi tyissuedExecut iveOr derNo.93- 01creatingTask


For ceKal i
kasant ocombat“ ill
egall oggi ng,l ogsmuggl i
ngorpossessi onofand/ ort
ranspor tof
i
llegal l
ycutorpr oducedl ogs, lumber ,flitchesandot herf orestpr oduct s”i nt hatcity
.Thet eam was
composedofper sonneloft hePhi l
i
ppi neAr my,PNP,DENRandt heOf ficeoft heCit
yMay orof
But uan.RespondentOdelBer nardoLausa,whowast heact ingchi efofci vi
l
iansecurityi nthe
may or’
sof fice,wasamemberoft het eam.OnJul y1, 1993, themember soft hetaskfor
cer eceived
conf identialinformat iont hattwot ruckl oadsofi l
legallycutl umberwoul dbebr oughttoBut uanCi t
y
from t heAmpay on-Tagui be-Tiniwisan ar ea.Accor dingly,t het eam setup acheckpoi ntal ong
kilomet er4i nBaan,But uanCi ty.Ther af teruponcat chingupwi tht het wocar sinthel att
er’
s
compound,t hecar etakeroft hecompoundwasnotabl et opr oduceanydocument sprov ingthe
l
egal ityofpossessi onoft hef orestpr oduct s.DENRof f
icerst hensei zedt het r
uckandlumber ,and
sincet her ewer enocl aimant saf terpost ingt henot iceofconf iscation,i twasdeemedf or feit
edin
fav oroft hegov ernment .
2mont hsaf terthesai df orf
ei tur e,pet it
ionerher ei
nf il
edasui tf orr epl
evintorecov erthe
trucksandl umber ,towhi chr espondentLausaf il
edamot i
onf orappr ov alofcount erbondand
dismi ssaloft her eplevinsi ncet hesei zur ewaspur suantt ot heRev i
sedFor estryCode.Tr i
alcourt
forpet it
ioner ,butwasr eversedi nt heCAi nf avorofLausa.Hencet hispet ition.

I
SSUE:WONt heRegionalTri
alCourtcoul
dinfactt
akecogni
zanceofther
eplev
insuit
,consi
deri
ng
thatt
heobjectwasther ecover
yofl umbersei
zedandfor
fei
tedbylawenfor
cementagentsofthe
DENRpursuanttoP.D.No.705( Rev
isedForest
ryCode)
,asamendedbyExecuti
veOrderNo.277.–
NO.

RATI O:Ther uleist hatapar tymustexhaustal ladmi nist r


ativeremedi esbef orehecanr esorttot he
cour t
s.I nal ongl i
neofcases,wehav econsi stentlyhel dt hatbef oreapar tymaybeal l
owedt o
seekt heinterventionoft hecour t,itisapr e-condi ti
ont hatheshoul dhav eav ai l
edhi msel fofal lthe
meansaf fordedbyt headmi nistr ati
vepr ocesses.Hence,i far emedywi thint headmi nistrati
ve
machi nerycanst i
llber esor tedt obygi vingt headmi nistrativeoffi
cerconcer nedev eryoppor tunit
y
todeci deonamat tert hatcomeswi thinhi sj urisdictiont hensuchr emedyshoul dbeexhaust ed
fi
rstbef orea cour t
’sj udicialpowercan besought . Thepr emat urei nvocat i
on ofa cour t’
s
i
nter v
entionisf ataltoone’ scauseofact ion.Accor dingl y,absentanyf i
ndingofwai v erorest oppel ,
thecasei ssuscept i
bleofdi smi ssal forlackofcauseofact i
on.
Aspet iti
onercl earlyf ai l
edt o exhaustav ailableadmi nistrati
ver emedi es,t heCour tof
Appeal scorrectlysetasi det heassai l
edor der soft het rialcour tgrantingpet i
tioner’sappl icati
onf or
arepl evi
nwr itanddeny ingpr i
vat er espondent ’smot i
ont odi smiss.Hav ingbeenf or feit
edpur suant
toP. D.No.705,asamended,t hel umberpr oper lycameundert hecust odyoft heDENRandal l
actionsseekingt or ecov erpossessi ont her eofshoul dbedi rectedtot hatagency .
Theappel latecour t’sdi rect i
vet ot hetr i
alcour tj udget oal l
owt her espondentagentoft he
DENRt of i
leacount er bondi nor dert or ecov ercust odyoft helumbershoul dbedi sregardedas
beingcont r
aryt oitsor dertodi smi sst her eplev i
nsui tofpet it
ioner.For ,i
ndeed, whati tshoul dhav e
donewast odi smi sst hecasewi thoutpr ejudicet opet i
tionerf i
li
nghercl aim bef oret heDENR.

PI
COPV.BASEMETALS
G.
R.No.163509      

ATTY.VI
LLEGAS 15
I
NTERNATI
ONALENVI
RONMENTALREGULATI
ONS
ANDNATURALRESOURCESDI
GESTS

December6,2006
Ponent
e:Ti
nga

FACTS:I n1987,t heCent ralMi ndanaoMi ni ngandDev el


opmentCor por at ion( CMMCIf orbr ev ity
)
ent eredi ntoaMi nesOper ati
ngAgr eementwi thBanahaw Mi ningandDev elopmentCor por ation
wher ebyt helat teragr eedt oactasMi neOper atorf ort heexpl or ation,dev elopment ,andev ent ual
commer cialoper ationofCMMCI 's18mi ningcl aimsl ocat edi nAgusandelSur .Pur suantt ot he
termsoft heAgr eement ,Banahaw Mi ningf iledappl icat i
onsf orMi ni ngLeaseCont ract sov ert he
mi ningcl aimswi tht heBur eauofMi nes.OnApr i
l29,1988,BanahawMi ni ngwasi ssuedaMi nes
Tempor ar yPer mi taut hor i
zi ngi tt oext ractanddi sposeofpr eciousmi ner al sf oundwi thini tsmi ning
claims.Si nceapor t
ionofBanahaw Mi ning' smi ningcl aimswasl ocat edi npet iti
onerPI COP' s
l
oggi ngconcessi oni nAgusandelSur ,BanahawMi ningandpet it
ionerPI COPent eredi ntoaMOA
wher ebypet itionerPI COP al lowedBanahaw Mi ninganaccesst oi tsmi ni ngcl ai ms.I n1991,
Banahaw Mi ni ng conv erted i tsmi ning cl aimst o appl i
cat i
onsf orMi ner alPr oduct i
on Shar ing
Agr eement s(MPSAf orbr ev i
ty).
Whi l
et heMPSA wer ependi ng,Banahaw Mi ning,on December18,1996,deci ded t o
sell/assi gni tsr i
ght sandi nterest sov er37mi ningcl aimsi nf av orofpr iv ater espondentBase
Met alsMi neral Resour cesCor por ati
on.Thet r
ansf eri ncl udedt hosecov er edbyi tsmi ni ngoper ating
agr eementwi th CMMCI .Upon bei ng i nf or med oft hedev elopment ,CMMCI ,ascl aim owner ,
i
mmedi atelyappr ov edt heassi gnmentmadebyBanahaw Mi ni ngi nf av orofpr i
vat er espondent
BaseMet als,ther ebyr ecogni zingpr ivat erespondentBaseMet alsast henewoper atorofi tscl aims.
OnMar ch10,1997,pr i
vat er espondentBaseMet alsamendedBanahaw Mi ni ng'spendi ngMPSA
appl icationswi t
ht heBur eauofMi nest osubst itutei tsel fasappl icantandt osubmi taddi tional
document sinsuppor toft heappl i
cat ion.Ar eacl ear ancesf rom t heDENRRegi onalDi rect orand
Super i
nt endentoft heAgusanMar shandWi l
dl i
feSanct uarywer esubmi tted, asr equired.
OnNov ember18, 1997, petitionerPI COPf il
edwi t
ht heMi nesGeo- Sci encesBur eau( MGB) ,
anOpposi ti
ont opr ivater espondentBaseMet als'appl icationbecausei tv iolat et henon- impai rment
clauseandwi llbepr ejudi cialt oher einpet itioner .ThePanelAr bitr
at ori nitiallyr uledf orpet itioner ,
butuponappealt ot heMi nesAdj udi cat ionBoar d,j udgmentwasi nf av orofr espondent ,CA
affirmedst atingt hatt hePr esident ialWar rant yofSept ember25,1968i ssuedbyt henPr esi dent
Fer dinandE.Mar cosmer el yconf ir
medt het i
mberl i
censegr antedt oPI COPandwar rant edt he
l
at ter'speacef ulandadequat epossessi onandenj oymentofi tsconcessi onar eas.I twasonl y
givenupont her equestoft heBoar dofI nv est ment st oest abli
sht heboundar iesofPI COP' st imber
l
icense agr eement .The Pr esident ialWar r ant ydi d notconv ertPI COP' st imberl i
cense i nt oa
cont ractbecausei tdidnotcr eat eanyobl igat ionont hepar toft hegov er nmenti nf av orofPI COP.
Thus, thenon- impai rmentcl ausef i
ndsnoappl i
cation.

I
ssue:W/Ntheconcessi
onareaofpeti
ti
oneri
sclosedt
omi ni
ngactiv
itiesandt
hattheconversi
on
oftheagr
eementint
oMPSAwi l
lruncount
ertothenon-
impair
mentcl auseoftheConst
itut
ion.–
NO,aTimberl
i
censeagreementi
snotacontract
,butamerepri
vi
lege.

RATI O:Weshoul dstat eatt hisjunct uret hatthepol i


cyofmul tiplel andusei senshr i
nedi nourl aws
towar dst heendt hatt hecount ry'
snat uralresour cesmayber ational l
yexpl ored,developed, util
i
zed
andconser ved.I nlikemanner ,RA 7942,r ecogni zingt heequi ponder ancebet weenmi ningand
ti
mberr ights,gi vesami ningcont ractort her ightt oent erat imberconcessi onandcutt i
mber
thereinpr ovidedt hatt hesur f
aceownerorconcessi onaireshal lbepr oper l
ycompensat edf orany
damagedonet ot hepr oper tyasaconsequenceofmi ningoper at i
ons.
First
ly ,
assumi ngt hatthear eacov eredbyBaseMet als' MPSAi sagov ernmentr eser vati
on,
definedaspr oclai
medr eservedlandsf orspeci fi
cpur posesot hert hanmi neralreservat i
ons,such
doesnotnecessar i
lypr ecludemi ningact i
vit
iesi nt hear ea.Sec.15( b)ofDAO96- 40pr ov idest hat
gov er
nmentr eservationsmaybeopenedf ormi ningappl i
cat ionsuponpr iorwr i
tt
encl ear anceby
thegov er nmentagencyhav i
ngjurisdictionov ersuchr eser vation.Sec.6ofRA7942al sopr ovi
des
thatmi ni ngoper ati
onsi nr eservedl andsot hert hanmi neralr eser vationsmaybeunder takenbyt he
DENR, subj ectt ocertai nlimitat
ions.Secondl y ,RA7942doesnotdi sallowmi ni
ngappl i
cat ionsi nall
forestr eser vesbutonl yt hosepr oclaimedaswat er shedf orestr eserv es.Therei snoev idencei n
thi
scaset hatt heareacov eredbyBaseMet als'MPSAhasbeenpr oclaimedaswat er shedf orest

ATTY.VI
LLEGAS 16
I
NTERNATI
ONALENVI
RONMENTALREGULATI
ONS
ANDNATURALRESOURCESDI
GESTS

reserves.DENRMemor andum Or derNo.03- 98,whi chpr ovidest heguideli


nesint heissuanceof
areast atusandcl earanceorconsentf ormi ni
ngappl i
cationspur suanttoRA7942,pr ovidesthat
ti
mberorf or estlands,mi li
tar
yandot hergov ernmentr eservati
ons,forestreservat
ions,f or
est
reservesot hert hancr iti
calwat ershedf orestr eserves,andexi sti
ngDENRPr ojectAr easwi thi
n
ti
mberorf or estlands,reservati
onsandr eserves,amongot hers,areopent omi ni
ngappl icati
ons
subjecttoar east atusandcl ear
ance.
Last ly,PICOP f ail
edt opr esentanyev idencet hatt hear eacov er
edbyt heMPSA i sa
protectedwi ldernessar eadesi gnatedasani ni ti
alcomponentoft heNIPASpur suantt oal aw,
presidential decree, pr
esidenti
alproclamat ionorexecut iveor derasr equi
redbyRA7586.

ASAPHI
LV.TUASON
G.R.NO.134030      
Apri
l25,2006

ATTY.VI
LLEGAS 17
I
NTERNATI
ONALENVI
RONMENTALREGULATI
ONS
ANDNATURALRESOURCESDI
GESTS

Ponent
e:Aust
ri
a-Mar
ti
nez

FACTS:OnMar ch24,1975,r espondentVi cent eTuason,Jr .ent eredi nt oaCont ractf orSal eand
Pur chaseofPer liteOr ewi thI ndupl exwher einI ndupl exagr eedt obuyal ltheper l
it
eor et hatmaybe
foundandmi nedi nTuason’ smi ningcl ai
ml ocat edi nTay sa,Dar aga,Al bay .I nexchange,I ndupl ex
wi l
lassi stTuasoni nsecur i
ngandper fectinghi srightov ert hemi ningcl aim .Ther eafter ,Tuason
execut edanAgr eementt oOper ateMi ningCl aimsi nf avorofpet iti
onerAsaphi lConst ruct ionand
Dev elopmentCor por ation.Lat er ,Tuasonf i
ledwi ththeBur eauofMi nes, DENRacompl aintagai nst
Asaphi land I ndupl exf ordecl arat ion ofnul lityoft he said Cont r
act s.Tuason al l
eged i n hi s
compl aintt hatt hest ockhol der sofI ndupl exf ormedandor gani zedI bal onMi neralResour ces,an
ent i
tywhosepur posei st omi neanyandal lki ndsofmi ner als,thatt hisi si nv i
olat i
onoft he
condi tioni mposedbyt heBoar dofonI ndupl exi ni tsJoi ntVent ureAgr eementwi thGr efco,I nc,
prohi biti
ngI ndupl exf rom mi ningper lit
eor e, t
hr oughanoper atingagr eementoranyot hermet hod;
thatI ndupl exacqui r
edt hemaj orityst ocksofAsaphi landt hat95% ofI balon’sshar eswer eal so
transf erredt oVi rgilioR.Romer o,whoi sast ockhol derofI ndupl ex,Asaphi landI balon.Tuason
claimed t hatsai d act s adv ersely af f
ect ed,notonl y hi si nterestas cl aimowner ,butt he
gov ernment ’sint erestaswel l.
Asaphi lf i
ledi tsAnswer ,pr ay ingf ort hedi smi ssaloft hecompl ai ntont hegr oundt hatt he
DENRhasnoj ur isdict i
onov ert hecase.I ndupl exf il
edaMot i
ont oDi smi sst hecompl aint ,alsoon
groundofl ackofj urisdi ct
ion.I ndupl excont endedt hatt of allwi t
hint hej ur
isdictionoft heDENR,
thecont rov er syshoul di nv ol
v eami ningpr oper tyandt hecont endingpar ti
esmustbecl ai mhol ders
and/ ormi ni ngoper ator s;andt hatt hedi sput ei nt hiscasei nv olves“ mi neralproduct ”andnota
mi ningpr oper ty,andt hepr otagoni st sar ecl aimhol der s(Tuason)andabuy er( Indupl ex).DENR
affirmed,butt he Mi nes Adj ucat ion Boar dr ev ersed,st ating t hatt he compl ainti sf ort he
cancel l
ationandr ev ocat i
onoft heAgr eementt oOper ateMi ningCl aims,whi chi swi thint he
j
ur i
sdi cti
onoft heDENRunderSect ion7ofPD1281.TheMABal sof oundt hatt heacqui siti
onby
Indupl exoft hemaj orityst ocksofAsaphi l,andI ndupl ex’sassumpt ionoft hemi ningoper ation
violatedt heBOIpr ohi bition.

I
ssue:W/ NtheDENRhasjuri
sdicti
onoverTuason’
scompl ai
ntf
ortheannul
mentoft heContr
act
forSaleandPur
chaseofPerl
i
teOr ebet
weenTuasonandI ndupl
ex,andt
heAgreementtoOperate
MiningClai
msbetweenTuasonandAsaphil;andsecond,WONt heMABer r
edininvali
dat
ingthe
AgreementtoOper
ateMini
ngClaims.–YES.

RATIO:I nsev er alcasesonmi ningdi sputes,t heCour trecognizedadi stincti


onbet ween( 1)t he
pri
mar ypower sgr antedbyper ti
nentpr ovisionsofl aw tot het henSecr etaryofAgr i
cultureand
NaturalResour ces( andt hebur eaudi r
ectors)ofanexecut iv
eoradmi nistrat
ivenat ure,suchas
granti
ngofl icense,per mi ts,leaseandcont racts,orappr oving,rejecting,r einstat
ingorcancel ing
appli
cat i
ons, ordeci dingconf l
ictingappl i
cations, and( 2)cont r
oversiesordi sagreement sofci vilor
contractualnat ure between l itigant s which ar e quest i
ons ofa j udi cialnat uret hatmaybe
adjudicatedonl ybyt hecour tsofj ust i
ce.Theal l
egat ionsi nTuason’ scompl aintdonotmakeouta
casef orami ningdi sput eorcont rov ersywi thint hejur i
sdicti
onoft heDENR.Whi l
et heAgr eement
toOper ateMi ningCl aimsi sami ningcont r
act ,thegr ounduponwhi cht hecont r
actissoughtt obe
annulledisnotduet oAsaphi l’sr efusalt oabi debyt het ermsandcondi tionsoft heagr eement ,but
duet oI ndupl ex’ sal l
egedv iolationoft hecondi ti
oni mposedbyt heBOIi ni t
sJoi ntVent ure
Agreementwi thGr efco, Inc..Al so, Tuasonsoughtt henul l
it
yoft heCont ractf orSaleandPur chase
ofPer l
iteOre, basedont hesameal l
egedv i
ol ati
on.Obv iousl
y ,t
hisr ai
sesaj udi
cialquest i
on, whi ch
i
spr operf ordet er
mi nationbyt her egul arcour t
s.
TheDENRi snotcal l
edupont oexer cisei tst echnicalknowl edgeorexper t
iseov erany
miningoper at ionsordi sput e;rat her ,itisbei ngaskedt odet erminet hev ali
dityoft heagr eement s
basedonci rcumst ancesbey ondt her espect i
v er i
ghtsoft hepar t
iesundert hetwocont racts.Thus,
the DENR Regi onalExecut iv e Di rect orwas cor recti n di
smi ssing t he compl aintf orl ack of
j
urisdicti
on ov erTuason’ s compl aint;consequent ly ,the MAB commi tted an er rori nt aking
cognizanceoft heappeal ,andi nr ul ingupont hev ali
dityoft hecont racts.
DIDI PIOEARTHSAVERSV.GOZUN
G.R.No.157882      

ATTY.VI
LLEGAS 18
I
NTERNATI
ONALENVI
RONMENTALREGULATI
ONS
ANDNATURALRESOURCESDI
GESTS

March30,2006
Ponent
e:Chico-
Nazar
io

FACTS:I n1987,Cor ypr omul gatedEO 279whi chempower edDENR t ost ipul atewi thf or eign
compani esr egar dingt echnicalorf inanciall argescal eexpl orationormi ning.I n1995,Ramos
signedi ntol awRA7942ort hePhi li
ppi neMi ningAct .In1994, Ramosal readysi gnedanFTAAwi th
ArimcoMi ningCo,anAust r
ali
ancompany .TheFTAAaut hor i
zedAMC( l
aterCAMC)t oexpl ore
37,000hect ar esofl andinQui ri
noandNuev aVi zcayaincludingBr gy.Didipi
o.Af tert hepassageof
thelaw, DENRt heni ssueditsImpl ement ingRul esandRegul ations.
Inseeki ngt onul l
ifyRep.ActNo.7942andDAO 96- 40asunconst itutional,pet i
tioner s
reasonedt hatt hesei neffectallowt heunl awf ulandunj ust“taking”ofpr opertyf orpr i
vatepur pose
i
ncont rav ent ionwi thSection9, Ar ti
cleI I
Ioft he1987Const itut i
on, mandat i
ngt hatpr ivatepr oper t
y
shallnotbe t aken exceptf orpubl i
c use and wi tht he cor r
esponding pay mentof j ust
compensat i
on.  They asser tt hatpubl icr espondentDENR,t hrough the Mi ning Actand i ts
Implement ingRul esandRegul ations,cannot ,oni tsown,per mi tent r
yintoapr ivatepr oper tyand
all
owt aki ngofl andwi t
houtpay mentofj ustcompensat ion.
Publ icr espondent sont heot herhandav ersthatSect ion76i snotat aki ngpr ovisionbuta
vali
dexer ciseoft hepol i
cepowerandbyv i
rtueofwhi ch,thest atemaypr escr iber egulati
onst o
promot et heheal th,mor als,peace,educat ion,goodor der,saf etyandgener alwel fareoft he
people. Thi sgov ernmentr egulationi nvolvest headj ustmentofr ightsforthepubl i
cgoodandi t
endeav orspot entialfortheuseoreconomi cexpl oit
ati
onofpr i
v ateproperty. Publ icrespondent s
concludedt hat“ tor equir
ecompensat i
oni nal lsuchci r
cumst anceswoul dcompelt hegov er
nment
toregulat ebypur chase.”

I
ssue:
W/NRA7942andt
heDENRRRsar
eval
i
d.–YES.

RATIO: TheSCnot edt her equisit


esofemi nentdomai n.Theyar efoll
owi ng:
(1)t heexpr opriatormustent erapr i
v atepr operty;
(2)t heent rymustbef ormor et hanamoment aryperi
od.
(3)t heent rymustbeunderwar rantorcol oroflegalauthorit
y;
(4) t hepr opertymustbedev ot edt opubl i
cuseorot herwisei nf ormal l
yappr opri
atedor
i
njuriouslyaf f
ect ed;
(5)t heut i
l
izati
onoft hepr oper tyf orpubl icusemustbei nsuchawayast oousttheowner
anddepr i
vehi m ofbenef icial enj oy mentoft heproperty.
Int hecaseatbar ,Didipiof ai l
edt oshow t hatthelaw i sinval id.I ndeedt hereistaki
ng
i
nvolvedbuti ti swithj ustcompensat ion.Sec.76ofRA7942pr ovidesf orj ustcompensat ionas
well
assect ion107oft heDENRRR.Towi t:
Sect ion76.xxxPr ov
ided,t hatanydamaget ot hepr opertyoft hesur f
aceowner ,
occupant ,orconcessi onaireasa consequenceofsuch oper ationsshal lbepr operl
y
compensat edasmaybepr ovidedf ori nt hei mplement i
ngr ul
esandr egul ati
ons.
Sect ion107.Compensat i
onoft heSur faceOwnerandOccupant -Anydamagedonet o
thepr opertyoft hesur f
aceowner s, occupant ,
orconcessi onair
et her eofasaconsequence
oft he mi ni
ng oper ati
ons oras a r esultoft he const r
ucti
on ori nstall
ation ofthe
infr
ast ructurement ionedin104abov eshal lbepr operl
yandj ustl
ycompensat ed.
Fur ther,mi ningi sapubl icpol icyandt hegov ernmentcani nv okeemi nentdomai nto
exer
ciseent ry,acquisiti
onanduseofpr iv atel ands.

PEOPLEVSROSEMOOR
G.
R.No.149927      

ATTY.VI
LLEGAS 19
I
NTERNATI
ONALENVI
RONMENTALREGULATI
ONS
ANDNATURALRESOURCESDI
GESTS

March30,2004
Ponent
e:Pangani
ban

FACTS:RosemoorMi ni
ngAndDev el
opmentCor por ationCor porat
ion( Rosemoor ),af terhav i
ng
been gr anted permi ssion t o prospectf ormar bl
edeposi tsi nthemount ai
nsofBi ak-na- Bato,
Bulacan,succeededi ndi scoveringmar bledeposi tsofhi ghqual it
yandi ncommer ci alquant it
ies.
Theyappl i
edwi ththeBur eauofMi nes( nowMi nesandGeosci encesBur eau),forthei ssuanceof
thecor respondinglicenset oexpl oitsaidmar bledeposi ts,whi chwasi ssuedt ot hem,gi vi
ngt hem
therightt oquar r
y330hect ar
esofl and.Af terErnest oMacedawasappoi ntedMi nisterofDENR, he
cancel l
edRosemoor ’sl i
cense.Rosemoorf i
ledf orinjunct iv
er eli
effrom theRTC, whichr ul edi ntheir
favor.
TheRTCsai dt hatRosewood’ sr espondent s’licensehadal readyr i
penedi ntoapr operty
ri
ght,whi chwaspr otect edundert heduepr ocesscl ause,andsuchr i
ghtwassupposedl yv iol
ated
whent helicensewasunj ustifi
ablycancel ledwi thoutnot i
ceandhear ing.Pet i
ti
onersav ert hatthe
l
icensecont ravenesPD 463becausei texceedst hemaxi mum ar eat hatmaybegr ant edt oa
Licenseef orquar ry
ing( 100hect ares),whi chr ender st hel icensev oi
d.TheCAsust ainedt heRTC
decisionbecauset hel i
censewasembr acedbyf our( 4)separ ateappl i
cat i
ons,andt hatt he100
hectarel i
mi t
ationwassuper sededbyRA7942.TheCAal sosai dt hatPr oclamat i
on84,whi ch
confirmedt hecancel lati
onoft helicense, impairedt henon- impai r
mentcl auseofcont ract s,abi l
l of
attai
nderandanexpostf actolaw.

I
SSUE:
W/NRosemoor
’sl
i
censewasv
ali
dlycancel
l
ed.–YES.

RATI
O:
Validi
tyofLi cense
PD 463,as amended,per tained t ot he ol d syst em ofexpl orati
on,dev el
opmentand
uti
li
zati
onofnat uralresour cest hroughl icenses,concessi onsorl eases,butwasomi ttedi nt he
1987Const it
utionasi twasdeemedv i
ol ati
veofi t
spr ov i
sions.Thi swasr epl
acedbyRA7942ort he
Phil
ippineMi ningActof1995r epealedoramendedal llaws, executi
veor der
s,pr esident
ialdecr ees,
rul
esandr egulations- -orpar tsthereof- -thatareinconsi stentwi thit.
Whi l
eRA7942hasexpr esslyr epeal edprovisionsofmi ninglawst hatarei nconsistentwi th
i
tsown,i tnonet helessr espect spr ev iouslyissuedv alidandexi st
ingl i
censes.I nt hi
scase,t he
ter
msofRosemoor ’
sl i
censewassubj ectt oPD463,t heexi stinglaw wheni twasgr ant ed.And
undersuchl aw,i tiscleart hatal icenseshoul donl ycov er100hect areswi thoutexcept ionsor
considerationtot henumberofappl i
cat ions.Thei ntentoft hel awwoul dbebr azenlycir
cumv ented
byruli
ngt hatal icensemaycov eranar eaexceedi ngt hemaxi mum byt hemer eexpedi encyoff ili
ng
severalappl i
cations.Suchr ulingwoul di ndir
ectl
yper mi tanactt hatisdirectl
ypr ohibit
edbyt hel aw.

Vali
dityofPr oclamat ionNo.84
Rosemoor ’sli
censemayber evokedorr escindedbyexecut iveact ionwhent henat i
onal
i
nt erestsor equires,becausei tisnotacont ract,proper tyorapr oper tyrightpr otect edbyt hedue
processcl auseoft heConst i
tution.Thi scondi ti
ont ot hel icensewasacknowl edgedbyRosemoor
i
ni tsper mi t
.Mor eov er,
gr anti
ngt hatRosemoor ’li
censei sv ali
d,i
tcanst i
llbev al
idlyr ev okedbyt he
Statei nt heexer ciseofpol icepower .Theexer ciseofsuchpowert hroughPr oclamat ionNo.84i s
clear l
yinaccor dwi t
ht her egal iadoct r
inewhi chr eser vest otheSt ateowner shipofal lnat
ur al
resour ces.
Pr oclamat ion84doesnoti mpai rthenon- i
mpai rmentcl ausebecauset hel icensei snota
cont ract.Eveni fthel i
censewer e,itisset t
ledthatpr ov isionsofexi stingl awsandar eservati
onof
policepowerar edeemedr eadi ntoi t
, becausei tconcer nsasubj ectimpr essedwi t
hpubl i
cwel f
ar e.
PN 84i sal sonotabi llofat tainderbecauset hedecl ar ationthatt hel icensewasv oidi snota
puni shment .Itisal sonotanexpostf act olawbecauset hepr ocl
amat iondoesnotf allunderanyof
theenumer atedcat egoriesofanexpostf actolaw.Andanexpostf actol awi slimi tedi nitsscope
onlyt omat t
er scriminalinnat ure.
LABUGALB’ LAANTRI BALASSOCI ATI ONI NC.,ETAL.V.RAMOS
G.R.No.127882
27Januar y2004

ATTY.VI
LLEGAS 20
I
NTERNATI
ONALENVI
RONMENTALREGULATI
ONS
ANDNATURALRESOURCESDI
GESTS

Ponent
e:Car
pio-
Mor
ales

FACTS:OnJul y25,1987,t henPr esi dentCor azonC.Aqui noi ssuedExecut iv


eOr der( E.O.)No.279
author izingt heDENRSecr etar yt oaccept ,consi derandev aluat epr oposal sf rom f oreign- owned
corpor ationsorf oreigni nv estor sf orcont ract soragr eement si nvol vingei thert echni calorf i
nanci al
assi stance f orl arge- scal e expl orat ion,dev elopment ,and ut il
izat i
on ofmi ner als,whi ch,upon
appr opr i
at er ecommendat ion oft he Secr etary,t he Pr esidentmay execut e wi t hthe f oreign
proponent .
OnMar ch3,1995,t henPr esi dentFi delV.Ramosappr ov edR. A.No.7942t o" gov ernt he
expl oration,dev elopment ,ut ilizat i
on and pr ocessi ng ofal lmi ner alr esour ces. "R. A.No.7942
definest hemodesofmi ner alagr eement sf ormi ningoper ations,out l
inest hepr ocedur ef ort heir
fi
li
ngandappr ov al,assi gnment /transf erandwi thdrawal ,andf ixest heirt erms.Si mi l
arpr ov i
sions
gov ernf inanci al ort echni cal assi stanceagr eement s.
OnApr il9, 1995, 30day sf ollowi ngi tspubl i
cat iononMar ch10, 1995i nMal ay aandMani l
a
Times,t wo newspaper sofgener alci rcul ation,R. A.No.7942 t ookef fect.Shor tl
ybef oret he
effect i
vityofR. A.No.7942,howev er ,oronMar ch30,1995,t hePr esidentent eredi nt
oanFTAA
withWMCPcov er i
ng99, 387hect ar esofl andi nSout hCot abat o, Sul tanKudar at, Dav aodelSurand
Nor thCot abat o.
OnAugust15,1995,t henDENRSecr etar yVi ctorO.Ramosi ssuedDENRAdmi ni strativ
e
Order( DAO)No.95- 23,s.1995,ot her wi seknownast heI mpl ement i
ngRul esandRegul ationsof
R.A.No.7942.Thi swasl aterr epeal edbyDAONo.96- 40, s.1996whi chwasadopt edonDecember
20, 1996.  
On Januar y10,1997,counsel sf orpet i
tioner s senta l et t
ert ot he DENR Secr et ary
demandi ngt hatt heDENRst opt hei mpl ement ati
onofR. A.No.7942andDAONo.96- 40, giv i
ngt he
DENRf ifteenday sf rom r ecei ptt oactt her eon.TheDENR,howev er,hasy ett or espondoracton
petitioner s'let ter .
Pet itioner scl aimt hatt heDENRSecr etaryact edwi thoutori nexcessofj urisdicti
on.
Theypr ayt hatt heCour tissueanor der :
(a)Per manent lyenj oi ningr espondent sf rom act ingonanyappl icat ionf orFi nanci alorTechni cal
Assi stanceAgr eement s;
(b)Decl aringt hePhi lippi neMi ningActof1995orRepubl icActNo.7942asunconst it
utionaland
null andv oid;
(c)Decl ar i
ngt heI mpl ement ingRul esandRegul ationsoft hePhi lippineMi ningActcont ainedi n
DENR Admi nist rative Or der No.96- 40 and al lot her si mi lar admi nistrat ivei ssuances as
unconst itutional andnul l andv oid; and
(d)Cancel ling t he Fi nanci aland Techni calAssi stance Agr eementi ssued t o West ern Mi ning
Philippines, Inc.asunconst itutional ,illegal andnul landv oi d.
InJanuar y2001,MMC–apubl iclyl i
stedAust ral i
anmi ni ngandexpl orat ioncompany–
soldi tswhol est akei nWMCPt oSagi t
t ariusMi nes, 60%ofwhi chi sownedbyFi li
pinoswhi l
e40%of
whi chi sownedbyI ndophi lResour ces,anAust raliancompany .DENRappr ov edt het r
ansf erand
regist r
ationoft heFTAAi nSagi t
tar ius’namebutLepant oConsol idat edassai l
edt hesame.WMCP
cont endst hatt heannul mentoft heFTAA woul dv iolateat reat ybet weent hePhi li
ppi nesand
Aust raliawhi chpr ovidesf ort hepr otect i
onofAust raliani nv estment s.

ISSUE:W/ N t
he Phili
ppi
ne Mini
ng Actis unconst
it
uti
onalf
oral
l
owi
ng f
ull
yfor
eign-
owned
corpor
ati
onstoexpl
oitPhi
li
ppi
neminer
alr
esources.–YES.

RATI O:RA7942ort hePhil


ippi
neMi ningActof1995i sunconst i
tutionalforpermitt
ingfull
yf orei
gn
ownedcor porati
onst oexploitPhil
ippinenaturalresources.Ar t
icleXI ISection2oft he1987
Const itut
ionr et
ainedtheRegal i
andoct r
inewhichst at
est hat“ Alllandsoft hepublicdomai n,
wat ers,mi ner
als,coal
,petrol
eum,andot hermineral
s,coal,petroleum,andot hermineraloi l
s,all
forcesofpot enti
alenergy,fi
sher
ies,forestsorti
mber ,wil
dli
fe,fl
or aandf auna,andot hernat ural
resour ces are owned by t he State.”The same sect i
on al so st ates t
hat,“explorat
ion and
dev el
opmentandut i
li
zati
onofnat ur
alresourcesshallbeundert hef ullcontr
olandsuper visionof
theSt ate.”

ATTY.VI
LLEGAS 21
I
NTERNATI
ONALENVI
RONMENTALREGULATI
ONS
ANDNATURALRESOURCESDI
GESTS

Conspi
cuouslyabsentinSect i
on2i st heprov
isi
oni nthe1935and1973Const ituti
ons
aut
horizi
ngt heStatetograntl i
censes,concessi
ons,orleasesf ort
heexplorat
ion,exploitat
ion,
devel
opmentorut i
li
zati
onofnat ur
alresour
ces.Ysuchomission,theuti
li
zat
ionofinal
ienablelands
ofpubl i
cdomai nthroughlicense,concessionorleasei sno longerall
owed undert he1987
Consti
tuti
on.

Undert heconcessi onsy stem, theconcessi onai remakesadi r ectequi t


yi nv est mentf ort he
purposeofexpl oit
ingapar ti
cul arnat ur alr esour cewi thinagi venar ea.Theconcessi onamount st o
compl et econt rolbyt heconcessi onai reov ert hecount ry’snat uralr esour ce, fori tisgi v enexcl usive
andpl enar yright st oexpl oi tapar t i
cul arr esour ceatt hepoi ntofext ract i
on.
The1987Const itut i
on,mor eov er ,hasdel et edt hephr ase“ managementorot herf ormsof
assist ance”i nt he1973Char ter .Thepr esentConst i
tutionnowal l
owsonl y“ techni calandf inanci al
assist ance. ”Themanagementoroper at i
onofmi ningact ivit
iesbyf oreigncont ractor s, t hepr i
mar y
featur eofser vi
cecont ract swaspr eci sel yt heev i
lt hedr aftersoft he1987Const i
t utionsoughtt o
avoi d.
Theconst itutionalpr ov isional lowi ngt hePr esidentt oent erint oFTAAsi sanexcept iont o
ther ulet hatpar t
icipat ioni nt henat ion’ snat ur alr esour cesi sr eser vedexcl usi velyt oFi l
ipinos.
Accor dingl ysuchpr ov isionmustbeconst ruedst rictl
yagai nstt heirenj oy mentbynon- Fil
ipinos.
Ther ef or eRA7942i si nv al i
di nsof arassai dactaut hor i
zesser vicecont r
act s.Al thought hest atute
empl oy st hephr ase“ fi
nanci alandt echni cal agr eement s”i naccor dancewi tht he1987Const ituti
on,
i
tsper t i
nentpr ov i
sionsact ual l
yt reatt heseagr eement sasser v i
cecont ract st hatgr antbenef icial
owner shi pt of or ei
gncont ract or scont rar yt ot hef undament al l
aw.
Theunder lyi
ngassumpt ioni nt hepr ov isionsoft hel aw i st hatt hef orei gncont ractor
managest hemi neralr esour cesj ustl i
ket hef or eigncont ractori naser v
icecont ract .Byal l
owi ng
foreigncont ractorst omanageoroper ateal ltheaspect soft hemi ningoper ation,RA7942hasi n
effectconv ey edbenef ici alowner shipov ert henat ion’smi neralr esour cest ot hesecont ractor s,
l
eav ingt heSt atewi thnot hi ngbutbar et i
tlet her eto.
Thesamepr ov isi ons,whet herbydesi gnori nadv ertence,per mitaci rcumv ent ionoft he
const itut ional l
y or dai ned 60- 40% capi talizat i
on r equi rementf orcor por ations orassoci ati
ons
engagedi nt heexpl oitation, dev el
opmentandut i
lizat i
onofPhi li
ppi nenat ural r
esour ces.
When par ts ofa st at ute ar e so mut ual ly dependentand connect ed as condi ti
ons,
consi der ati
ons,i nducement sorcompensat ionsf oreachot herast owar rantabel ieft hatt he
l
egi slat urei ntendedt hem asawhol e, theni fsomepar tsar eunconst i
tutional, al
l prov isi onst hatar e
thusdependent , condi tional orconnect edmustf al lwi ththem.
UnderAr ti
cleXI ISect ion2oft he1987Char ter ,foreignownedcor por ationsar el imi tedonl y
tomer el ytechni calorf inanci alassi stancet ot heSt atef orl argescal eexpl oration,dev elopment
andut ilizationofmi ner al s, petr oleum andot hermi ner aloils.

I
SSUE:
W/Nt
heFTAAbet
weenWMCPandt
hePgi
l
ippi
nesi
saser
vicecont
ract
.–YES.

TheFTAAbet weenWMCPandt hePhi li


ppinegov er nmenti sli
kewiseunconst i
tuti
onalsi ncet he
agreementi tselfi
sadev icecont ract .
Section1. 3oft heFTAAgr antsWMCP,af ullyforei
gnownedcor porati
on,t he“exclusive
ri
ghttoexpl ore,exploit,ut i
li
zeanddi sposeofal lminer alsandby -pr
oduct sthatmaybepr oduced
fr
om thecont ractarea.”Sect ion1. 2oft hesameagr eementpr ovi
dest hatWMCPshal lprovi
de“ al
l
fi
nancing,technol ogy,management ,andpersonnel necessar yfortheMi ningOper at
ions.”
Thesecont r
actualst ipulationsandr elat
edpr ovisi
onsint heFTAAt akent ogether,grant
WMCP benef i
cialowner shipov ernat uralresourcest hatproperlybelongt otheSt ateandar e
i
ntendedf ort hebenef i
tofi tscitizens.Thesest i
pulat i
onsar eabhorrentt othe1987Const i
tut
ion.
Theyar epr eciselythev icest hatt hef undament all aw seekst oav oi
d,t heev il
st hatitaimst o
suppress.Consequent l
y ,thecont ractf r
om whi chtheyspr i
ngmustbest ruckdown.

LABUGALB’
LAANTRI
BALASSOCI
ATI
ONI
NC.
,ETAL.V.RAMOS
G.R.No.127882.
December1,2004

ATTY.VI
LLEGAS 22
I
NTERNATI
ONALENVI
RONMENTALREGULATI
ONS
ANDNATURALRESOURCESDI
GESTS

Ponent
e:Pangani
ban

FACTS:Int heJanuaryDeci si
on,theCour tenbancpr omul gat
edi t
sDeci sion,grant i
ngt hePet i
tion
anddeclaringtheunconstit
uti
onalityofcer t
ainprov i
si
onsofRA7942( Phil
ippineMi ningAct )
,DENR
AO96- 40,aswel lasoft heent i
reFTAAexecut edbet weenthegov ernmentandWest er
nMi ning
Corp.Phili
ppines,anAust r
ali
ancor porat
ion.(WMCP) .TheCour tsai dRA7942ort hePhili
ppi ne
MiningActof1995andi tsimplement ingrulesareunconst i
tut
ionalforallowing“ servicecontracts”
nowpr ohibit
edbyt he1987Char ter.TheCour tsaidFTAAi saser vi
cecont r
actt hatgr ant
scont r ol
orbenefi
cialownershipoverthenat ion’smineralresour cestoforei
gncont r
actors,leav i
ngtheSt ate
wit
hnot hingbutbar et i
tlethereto.Itwasal soont hisgroundt hatt heCour tstruckdownas
consti
tuti
onallyi
nfi
rmt heFTAAbet weent hegov ernmentandWMCP.

ISSUE: Whet
herornotthephrase“Agr
eementsInvol
vingEit
herTechni
calorFi
nanci
alAssi
stance”
contai
nedinparagraph4ofSecti
on2ofAr t
icl
eXIIoft heConst
it
uti
onwaspr oper
lyi
nter
pretedin
theprevi
ousdecisi
ontowarr
anttheunconst
it
uti
onali
tyofRA7942andt heFTAAofWMCP.–YES.

RATI O:
ThePr operI nterpret ationoft heConst i
tut i
onalPhr ase“ Agr eement sI nvolvi
ngEi therTechni cal or
Financial Assist ance”
Sect i
on2Ar ti
cl eXIIdoesnotr ev ealanyi ntent iont oproscr i
bef oreigninv ol
vementi nt he
managementoroper ationofmi ningact i
v i
ti
esort oel i
mi nateser vi
cecont racts,nordoesi thav e
expr esspr ohi biti
ont ot hi
sef fect .Hadt hef ramer si nt endedt opr ohi bitdirectpar ti
cipationofan
aliencor por ationint heexpl or ationoft hecount ry’
snat ur alr esources, theywoul dempl oyedcl earl
y
rest r
icti
v el anguagebar ri
ngf or eigncor porati
onf rom di rect l
yengagi ngi nt heexpl orat i
onoft he
count ry’snat ural r
esour ces.
For eigncor por ati
onsmayi ndeedpar ti
cipat ei nt heexpl oitation,dev elopmentanduseof
Phi l
ippinenat uralresour cesbutsubj ectt otheful l
cont rol andsuper visionoft heSt ate.RA7942, i
ts
i
mpl ement ingr ul
es( DAO96- 40)andt heFTAAent er edi nt obyt henGov ernmentandWMCPgr ant
theGov er nmentf ullcont r
olandsuper visionov eral laspect sofpl annedexpl or
ation,dev elopment
andut il
i
zat ionact i
v i
t i
es.
Sect i
ons7. 8and7. 9oft heFTAAhowev erar eobj ecti
onabl eandv oidf orbeingcont r aryto
publ i
cpol i
cy .Sect ion7. 8permi t
st hesum spentbygov er nmentf ort hebenef itofthecont ract orto
bededuct i
bl ef r
om t heSt at e’sshar ei nt henetmi ning r evenuessi ncei tconst itutesunj ust
enr i
chmentont hepar toft hecont ract oratt hegov ernment ’
sexpense.Sect i
on7. 9,meanwhi le,
depr ives t he Gov er nmentofi ts shar ei nthe netmi ning revenues i nt he ev entt he f oreign
stockhol der sofaf or eignmi ni ngcompanysel l60%ormor eofthei requi tytoaFi li
pinoci tizenor
cor poration.
Thus,wi tht heexcept i
onofSect i
ons7. 8and7. 9oft hesubj ectFTAA,t heFTAA,RA7942
andDAO96- 40ar edecl aredconst itutional .

TheMeani ngof“ Agreement sI nv olv


ingEi therTechni cal orFi nanci alAssi stance”
A const itutionalpr ovi
sion speci fi
cally allowing f oreign-owned cor por ation tor ender
financialort echni calassi stancei nr espectofmi ningoranyot hercommer ci
alact i
vitywascl early
unnecessar y;thepr ov i
sionmeantt or efert omor et hanmer efi
nanci al ortechni cal assistance.
Thef r
amer soft heConst itution, dur i
ngi tsdeliberati
onr egar dingf oreigni nvest mentinand
managementofanent erpri
sef orlarge- scal eexpl orati
on,dev elopmentandut ili
zationofmi nerals
spokeaboutser v icecont ractsast heconceptwasunder stoodi nt he1973Const i
tut
ion.Iti s
obv i
ousf rom t heirdi scussionst hatt heydi dnoti ntendt obanorer adicateser v i
cecont racts.
Inst ead,theywer ei ntentoncr aftingpr ov isionst oputi npl acesaf eguar dst hatwoul del iminatethe
abusespr eval
entdur ingthemar ti
all awr egime.Theywer egoi ngt oper mi tser vicecont r
actswi t
h
for eigncor por ationsascont ractor s– butwi t hsaf etymeasur est o pr eventabuses– asan
except iont ot hegener alnorm est abl i
shedi nt hef ir
stpar agraphofSect ion2ofAr ti
cleXI I
,which
reser vesorl imi tst oFi l
ipi
noci t
izensandcor por ati
onst hatar eatl east60per centownedbysuch
citizenst heexpl orat ion,dev el
opmentandut il
izationofmi neralorpet roleum r esour ces.Thiswas
pr ompt edbyt heper ceivedinsuf f
iciencyofFi l
i
pi nocapi t
alandt hef el tneedf orf oreignexper ti
sein
theEDUofmi ner al resour ces.

ATTY.VI
LLEGAS 23
I
NTERNATI
ONALENVI
RONMENTALREGULATI
ONS
ANDNATURALRESOURCESDI
GESTS

Thedr afters,byspeci fy ingsuchagr eement sinv ol


v i
ngassi stance, necessar ilygav eimpl ied
assentt oev er
y t
hingt hatt heseagr eement sent ailedort hatcoul dr easonabl ybedeemednecessar y
tomaket hem tenabl eandef fect ive–i ncl udingmanagementaut hor i
tywi thr espectt ot heday -t
o-
dayoper ati
onsoft heent er prise,andmeasur esf orthepr otectionoft hei nt er estsoft hef oreign
cor poration,atleastt ot heext entt hatt heyar econsi stentwi thPhi l
i
ppi nesov er eigntyov ernatur al
resour ces,t heconst it
ut ionalr equirementofSt at econt rol,andbenef i
cialowner shi pofnat ur al
resour cesr emainsv est edi nt heSt ate.
I
ti scleart hatagr eement sinvol vi
ngei thert echni calorf inanci alassi st ancer efer r
edt oi n
par agraph4ar ei nf actser vicecont r
act s,butsuchnew ser vicecont r
actsar ebet weenf oreign
cor porationsact i
ngascont ract orsont heonehand, andont heot herhandgov er nmentaspr i
ncipal
or“ owner ”)oft hewor ks) ,wher ebyt hef oreigncont r
act orpr ov i
dest hecapi tal,technol ogyand
techni calknow- how,andmanager ialexper t
isei nt hecr eat i
onandoper ationoft hel arge-scale
mi ning/ext r
activeent erpr ise, andgov ernmentt hroughi tsagenci es( DENR, MGB)act ivelyexer cises
fullcont rolandsuper visionov ert heent ireent er prise.
Suchser v i
cecont ract smaybeent eredi nt oonlywi thr espectt omi ner aloils.Thegr antof
suchser v i
ceissubj ectt osev er alsaf eguar ds, amongt hem:( 1)t hatt heser vi
cecont ractbecr afted
i
naccor dancewi thagener allawset ti
ngst andar doruni f
ormt erms,condi t
ionsandr equi r
ement s;
(2)t hePr esi
dentbet hesi gnat oryf ort hegov er nment ;and( 3)t hePr esidentr epor tt heexecut ed
agr eementt oCongr esswi t hint hirt
yday s.

Ul ti
mat eTest :Ful lSt ateCont rol
Thepr i
macyoft hepr incipl eoft heSt ate’ssov er eignowner shipofal lmi ner alr esour ces,
andi tsf ul lcont rolandsuper visionov eral laspect sofexpl oration,dev el opmentandut ilizat ionof
nat uralr esour cesmustbeuphel d.But“ fullcont rolandsuper vision”cannotbet akenl iteral l
yt o
meant hatt heSt atecont r
ol sandsuper visesev ery t
hi ngdownt othemi nut estdet ail
s, andmakesal l
requi redact ions,ast hiswoul dr enderi mpossi blet hel egi ti
mat eexer cisebyt hecont ract orofa
reasonabl e degr ee ofmanagementpr er ogat ive and aut hor i
ty,i ndispensabl et ot he pr oper
funct ioni ngoft hemi ningent er prise.Al so,t hegov ernmentneednotmi cro- managet hemi ning
oper at i
onsandday -
to-dayaf fai rsoft heent erprisei nor dert obeconsi der edasexer ci singf ull
cont rol andsuper vision.
TheSt ate’sf ullcont r
olandsuper v isionov ermi ningoper ati
onsar eensur edt hrought he
differentpr ov isionsi nRA7942.Thegov ernmentagenci esconcer nedar eempower edt oappr ov eor
disappr ov et hev ariouswor kpr ogr amsandcor respondi ngmi nimum expendi tur ecommi tment sfor
eachoft heexpl or ati
on, dev elopmentandut ilizationphasesoft heent erprise.Oncet heyhav ebeen
appr ov ed,t he cont r
act or’
scompl iance wi thi tscommi tment st herein wi llbe moni tor ed.The
cont ract ori sal somandat edt oopeni tsbooksofaccount sandr ecor dsf orscr uti
ny ,toenabl et he
Stat et odet er mi nei ft hegov er nmentshar ehasbeenf ullypai d.TheSt atemayl i
kewi secompel
compl iance by t he cont ractor wi th mandat or yr equi rement s on mi ne saf et y,heal th and
env ironment alpr ot ection,andt heuseofant i
-pol l
ut i
ont echnol ogyandf acili
ties.Thecont ract oris
alsoobl igat edt oassi stt hedev elopmentoft hemi ningcommuni ty
,andpayr oy alti
est ot he
i
ndi genouspeopl esconcer ned.Andv i
ol ationofanyofFTAA’ st ermsandcondi tions,and/ or
noncompl iancewi thst atutesorr egul ations,maybepenal i
zedbycancel lati
onoft heFTAA.Such
sanct i
oni ssi gnificantt oacont ract orwhomayhav ey ett or ecov ert hetensorhundr edsofmi l
lions
ofdol l
ar ssunki nt oami ningpr oject .
Ov er al l
,theSt atedef i
ni tel
yhasapi v otalsayi nt heoper ationoft hei ndi vi
dualent er pr i
ses,
andcansetdi rect i
onsandobj ect i
v es,det ectdev iationsandnon- compl i
ancebyt hecont ract or; and
enf orcecompl ianceandi mposesanct ionsshoul dt heoccasi onar ise.Hence, RA7942andDAO96-
40v esti ngov ernmentmor et hanasuf f
icientdegr eeofcont rolandsuper v i
sionov ert heconductof
mi ningoper ations.
Sect i
on3( aq)ofRA7942wasobj ect edt oasbei ngunconst i
tuti
onalf oral lowi ngaf or eign
cont ract ort oappl yf orandhol danexpl orat ionper mi t
.Dur ingt heexpl or at
ionphase,t heper mit
grant ee( andpr ospect ivecont ract or)i sspendi ngandi nv estingheav i
lyi nexpl or at i
onact iviti
es
withouty etbei ngabl etoext ractmi neralsandgener ater evenues.Theexpl orationper mi ti ssued
underSect ion3( aq),20and23ofRA7942,whi chal lowsexpl orationbutnotext r
act ion,ser v est o
prot ectt hei nt erest sandr ightsoft heexpl or at ionper mi tgr antee( andwoul d-becont ract or ),for eign
orl ocal .Ot her wi se, t
heexpl orat ionwor ksal readyconduct ed, andexpendi t
uresal readymade, may

ATTY.VI
LLEGAS 24
I
NTERNATI
ONALENVI
RONMENTALREGULATI
ONS
ANDNATURALRESOURCESDI
GESTS

enduponl ybenef
it
ingcl ai
m- j
umpers.Thus,Sect
ion3( aq)ofRA7942isnotunconst ituti
onal
.
Theprovi
sionsoft heWMCPFTAA, farf
rom consti
tuti
ngasurrenderofcontrolandagr ant
ofbenef i
ci
alowner shipofmi neralresour
cestot hecont r
actori
nquest i
on,v esttheSt at
ewi th
controlandsupervisionov erpracti
call
yallaspectsoft heoperati
onsoft heFTAA cont ractor,
i
ncludingthechargingofpr e-
operat
ingandoper ati
ngexpenses,andt hedisposit
ionofmi neral
products.
Thereisli
kewi senor eli
nquishmentofcont r
olonaccountofspeci fi
cprov isi
onsoft he
WMCPFTAA.

TheFTAApr ov isionsdonotr educeorabdi cat eSt at econt rol: NoSur renderofFi nanci al Benef its
Thesecondpar agr aphofSect ion81ofRA7942hasbeendenouncedf oral legedl ylimi ting
theSt ate’sshar ei nFTAAswi thf oreigncont ract orst oj ustt axes, fees, anddut ies, anddepr i
v i
ngt he
Stat eofashar ei nt heaf ter -t
axi ncomeoft heent erpr ise.Howev er,t hei ncl usi onoft hephr ase
“amongot hert hings”i nt hesecondpar agr aphofSect i
on81cl ear l
yandunmi st akabl yr ev eal st he
l
egi sl at i
vei ntentt ohav et heSt at ecol lectmor et hanj ustusual taxes, dut i
esandf ees.
Thus,DAO99- 56,t he“ Gui del i
nesEst abl i
shi ngt heFi scalRegi meofFi nanci alorTechni cal
Assi st anceAgr eement s”spel lsoutt hef inanci albenef itsgov ernmentwi llr ecei vef rom anFTAA, as
consi st i
ngofnotonl yabasi cgov ernmentshar e,compr isedofal ldi rectt axes,f eesandr oy alties,
aswel lasot herpay ment smadebyt hecont ract ordur ingt het er m oft heFTAA,butal soan
addi tionalgov er nmentshar e,bei ngashar eint heear ni ngsorcashf lowsoft hemi ni ngent er prise,
soast oachi ev eaf ifty-fif tyshar ingofnetbenef itsf rom mi ni
ngbet weent hegov ernmentandt he
cont r act or.
Thebasi cgov er nmentshar eandt headdi tionalgov er nmentshar edonoty ett akei nt o
accountt hei ndi rectt axesandot herf i
nanci alcont ribut ionsofmi ningpr oject s,whi char er ealand
actualbenef i
tsenj oyedbyt heFi lipinopeopl e;i ft hesear et akeni nt oaccount ,t otalgov ernment
shar ei ncr easest o60per centorhi gher( asmuchas77per cent , and89per centi nonei nst ance)of
thenetpr esentv al ueoft ot al benef i
tsf rom t hepr oj ect .
Thet hirdorl astpar agr aphofSect i
on81ofRA7942i ssl ammedf ordef er r
ingt hepay ment
oft hegov ernmentshar ei nFTAAsunt ilaf t
ert hecont ract orshal lhav er ecov er edi tspr e-oper ating
expenses,expl or ationanddev elopmentexpendi t
ur es.Al legedl y ,thecol lect ionoft heSt at e’sshar e
i
sr ender eduncer tain,ast her ei snot imel i
mi tinRA7942f ort hisgr aceper iodorr ecov eryper i
od.
Butal thoughRA7942di dnotl imi tthegr aceper iod,t heconcer nedagenci es( DENRandMGB)i n
formul atingt he1995and1996I mpl ement ingRul esandRegul at i
onpr ov i
dedt hatt heper i
odof
recov er y, r
eckonedf rom t hedat eofcommer cialoper at i
on, shal lbef oraper i
odnotexceedi ngf ive
year s, orunt ilthedat eofact ual recov ery, whi chev ercomesear li
er .
Si nceRA 7942al l
egedl ydoesnotr equi regov er nmentappr ov alf ort hepr e- oper at ing,
expl or ationanddev elopmentexpensesoft hef orei gncont ract or s,iti sf ear edt hatsuchex penses
coul dbebl oatedt owi peoutmi ningr ev enuesant icipat edf or10y ear s,wi tht her esul tt hatt he
Stat e’sshar eiszer of ort hef irst10y ear s.Thear gumenti sbasedoni ncor recti nfor mat ion.Under
Sect ion23ofRA7942,t heappl icantf orexpl or ationper mi tisr equi r edt osubmi tapr oposedwor k
progr am f orexpl orat ion,cont aini ngay earlybudgetofpr oposedexpendi tures,whi cht heSt at e
passesuponandei therappr ovesorr eject s;ifappr ov ed, thesamewi l
l subsequent l
yber ecor dedas
pre- oper atingexpensest hatt hecont ractorwi ll hav et or ecoupov ert hegr aceper iod.
TheGov er nmenti sabl et oknow aheadoft imet heamount sofpr e-oper at ingandot her
expensest ober ecov er ed,andt heappr oximat eper i
odoft imeneededt her efor ebecauseunder
Sect ion24,whenanexpl orationper mitteef i
leswi t
ht heMGBadecl ar at i
onofmi ningpr oject
feasi bi li
ty,i tmustsubmi ta wor kpr ogr am f ordev elopment ,wi th cor respondi ng budget ,f or
appr ov albyt he Bur eau,bef ore gov ernmentmaygr antan FTAA orMPSA orot hermi ner al
agr eement s.Thegov er nmenthast heoppor tuni tyt oappr ov eorr ejectt hepr oposedwor kpr ogr am
andbudget edexpendi tur esf ordev elopmentwor ks,whi chwi llbecomet hepr e- oper atingand
dev el opmentcost st hatwi llhav et ober ecov er ed.
Mor eov er ,ther ei snoconcr etebasi sf ort hev iewt hat ,inFTAAswi thaf or eigncont ract or,
theSt atemustr ecei veatl east60per centoft heaf ter -taxi ncomef rom t heexpl oi tationofi ts
mi ner alr esour ces,andt hatsuchshar ei st heequi v alentoft heconst itut i
onalr equi rementt hatat
l
east60per centoft hecapi tal,andhence60per centoft hei ncome,ofmi ningcompani esshoul d
remai ni nFi l
i
pi nohands.Ev eni ft heSt at ei sent itledt oa60per centshar ef rom ot hermi ner al

ATTY.VI
LLEGAS 25
I
NTERNATI
ONALENVI
RONMENTALREGULATI
ONS
ANDNATURALRESOURCESDI
GESTS

agreements( CPA,JVAandMPSA) ,thatwoul dnotcr eateapar all


eloranal ogoussi t
uationf or
FTAAs..
TheChar t
erdi dnoti ntendt ofixani ron-cl
adr ul
eof60per centshar e,appl i
cablet oall
sit
uati
ons,r egardlessofci rcumst ances.Thet ermsandcondi tionsofpet r
oleum FTAAscannot
serveasst andar dsformi neralmi ningFTAAs, becausethet echnicalandoperationalr equir
ement s,
coststr
uctur esandi nvestmentneedsofof f
-shorepetrol
eum expl orati
onanddr ill
i
ngcompani esdo
nothav et he r emot estr esembl ance t ot hose of on-shor e mi ni
ng compani es.To av oid
compromi singt heSt ate’
sf ullcont r
olandsuper vi
sionov ertheexpl oit
ati
onofmi neralresources,
theremustbenoat temptt oi mposea“ mini
mum 60per cent”rule.Iti
ssuffi
cientt hattheSt atehas
thepowerandmeans,shoul ditsodeci de,togeta60per centshar e(orgreat er);andi tisnot
necessarythatt heSt atedoessoi nev erycase.

METROI
LOI
LOWATERDI
STRI
CTVSCA
G.R.No.122855      
March31, 2005
Ponente:Tinga

FACTS:Met
roI
loi
l
oWat
erDi
str
ict(
MIWD)
,whi
chwasgr
ant
edwat
err
ight
stoext
ractandwi
thdr
aw

ATTY.VI
LLEGAS 26
I
NTERNATI
ONALENVI
RONMENTALREGULATI
ONS
ANDNATURALRESOURCESDI
GESTS

groundwat erwi thi


ni t
sj uri
sdiction.MI WD f i
ledani njuncti
oncaseagai nstpr ivater espondent s
(Navaetal .)f
orwi t
hdrawinggr oundwat erwithintheformer ’
sjur
isdicti
onwi thoutsecur i
ngawat er
permi tfr
om NWRC, whichwasst il
lbeingdoneuponf i
lingthepetiti
on.Pr i
vater espondent sav err
ed
thattheRTChadnoj ur
isdict
ionov ert
hemat ter ,
asthecaseswer ewi t
hintheor igi
nal andex clusi
v e
j
urisdicti
onoft heNat i
onalWat erResour cesCounci l( WaterCounci l)
.TheRTC di smissedt he
petit
ionssay i
ngi thadnoj urisdi
ctionandt hatMWI Dfailedtoexhaustadmi nistrati
v eremedi es.The
CAaf fi
rmedtheRTCDeci sion,say ingthattheNWRChasj uri
sdi
ctiontohearanddeci dedisputes
rel
atingt oappropr i
ati
on,util
izat
ionandcont rolofwaterwhi chwast hesubj ectmat teroft hecase.

I
SSUE:
W/NNWRChadj
uri
sdi
cti
onov
ert
hecase.–NO.

RATI O:Thepet i
tionsf i
ledbef oretheRTCwer ef ort hei ssuanceofani njunct i
onor derf orpr i
vate
respondent st oceaseanddesi stfrom ext r
act ingorwi thdr awi ngwat erf rom MI WD’ swel landf rom
sell
ingthesamewi thini t
sser v i
cear eas.Thepet it
ionsf ocusont hev iolati
onsi ncur redbypr i
vate
respondent sbyv i
rtueoft heiral l
egedunaut hor izedext r
act i
onandwi thdrawalofgr oundwat er
withinpet i
tioner’sser vicear ea, vi
sa-a-vis MI WD’ sv est edr i
ght sasawat erdi stri
ct .Ati ssuei s
whet herornotpr ivaterespondent s’extractionandsal eofgr oundwat erwi thinpet i
tioner ’
sser vice
areav i
olatedpet it
ioner’srightsasawat erdi strict.
Thei nstantcasecer tainlycallsfort heappl i
cat ionandi nterpretat i
onofper ti
nentl awsand
j
ur i
sprudencei nor dertodet erminewhet herpr iv ater espondent s’actionsv i
olateMI WD’ sright sasa
wat erdi
strictandj ustif
yani njunction.Thi sissuedoesnotsomuchpr ovideoccasi ont oinv oket he
specialknowl edgeandexper ti
seoft heWat erCounci lasi tnecessi tatesj udicialint
erv ention.Whi l
e
i
nitial
lyitmayappeart hatt herei sadi mensi ont othepet i
tionswhi chper tainstot hespher eoft he
Wat erCounci l,i
nr eali
tythemat t
erisatmostmer elycol later altot hemai nt hrustofthepet i
tions.
MIWD had an appr oved Wat erRi ght s Gr antf rom t he Depar t
mentofPubl ic Wor ks,
Transpor t
ationandCommuni cations.Thet rialcour twasnotaskedt ogr antMI WDt her ightt ouse
butt ocompelpr i
vater espondent stor ecogni zet hatr ight.Thus,t het ri
alcour t
’sjurisdictionmust
beuphel dwher et hei ssuei nvolvedi snott heset t
lementofawat err ightsdisput e,butt he
enjoymentofar i
ghtt owat erusef orwhi chaper mi twasal readygr anted.

BFNORTHWESTHOMEOWNERSASSOCI
ATI
ON,
INC.VSI
ACANDBFHOMES,
INC.
G.R.No.72370
May29, 1987
Ponente:Melenci
o-Her
rer
a

FACTS:BFHomes,Inc.(HOMES) ,whi
chwasgrant
ed acerti
fi
cat
eofpubli
cconvenienceby
Nati
onalWat
erResources Counci
l(NWRC)[
for
merlythe Boar
d ofPowerand Waterworks]
,

ATTY.VI
LLEGAS 27
I
NTERNATI
ONALENVI
RONMENTALREGULATI
ONS
ANDNATURALRESOURCESDI
GESTS

i
ncr easedwat err atesbyaut hori
tyoft helatter.BFNor t
hwestHomeowner sAssoci ati
on,I nc
(ASSOCI ATI ON)filedapet iti
onenjoinBFHomes, I
nc.from collecti
ngf rom ASSOCIATIONmember s
theadj ustedwat err at
esf orbeingar bi
trar
yandunr easonableandt oannulNWRC’ sOr dergranting
HOMESaut horit
yt ochar get heincreasedwat erratesont hegr oundt hatitwasrenderedwi thout
procedur alduepr ocessandwi thoutorinexcessofj urisdi
cti
onandwi thgraveabuseofdi screti
on.
HOMESf il
edamot i
ontodi smissbutwasdeni ed.
Howev er,uponpet it
ionbyHOMES,t heCAr ev er
sedandhel dt hattheRTCwaswi thout
j
urisdicti
ont oent ertai
nthecasesi nceNWRC,whi cht ookov erthef uncti
onsofthePubl i
cSer vice
Commi ssion,hast herankofaRegi onalTri
alCour tandi t
sdeci si
ononwat erratesmayonl ybe
revi
ewedbyt heSupr emeCour t
.

I
SSUE:W/NtheRTChasjuri
sdi
cti
onoveract
ionstoannulOr
der
s,Resol
uti
onsand/
orDeci
sionsof
t
heNati
onal
WaterResour
cesCounci
l(NWRC)r el
ati
vet
owaterr
ates.–YES.

RATI O:UnderPD 1067,i texpl icitly states thatdeci si ons oft he NWRC on wat err ights
cont roversies maybeappeal edt ot heCour tofFi rstI nst ance.TheNWRC i st husr ankedwi th
"i
nf eri
orcour t
s,"whi ch,undert heSCI nter
im Rul esandGui delinesar elistedast heMet ropol i
tan
TrialCour ts,Muni cipalTr ialCour tsandMuni cipalCi rcui tTr ialCour t
s.Jur isdictionov eract ionsf or
annul mentofNWRCdeci sionslieswi ththeRegi onal Tr ial Cour t
s, par ti
cularly,whenwet akenot eof
thef actthatt heappel l
atej uri
sdi ctionoft heRegi onalTr ialCour tov erNWRCdeci si
onscov erssuch
broadandal lembr acinggr oundsasgr aveabuseofdi scr etion,quest i
onsofl aw,andquest ionsof
factandl aw.
Thedi stinctionmadebyt heCAbet ween" wat err ight scont rov ersi
es"whi chi tmai nt ainsar e
appeal ablet ot heRegi onalTr ialCour ts,and" waterr atesdi sputes"whi chi tsay sar eappeal ablet o
theCour tofAppeal s,isnotwel lt aken.Consi der i
ngt hatr ate-f i
xingi smer el yani nci dentt ot he
grantofacer t
ificateofpubl i
cconv eni ence,i
twoul dbei rregul arifdi sputesov erwat err ates, shoul d
behel dappeal ablet ot heCour tofAppeal swhi lecont rov er si
esov erwat err i
ght s,t hel atterbei ng
thepr ivi
legegr ant edbyt hegov er nmentt oappropr i
ateandusewat erand, ther efore, apr imar yright,
woul dbeappeal ableonl ytot heRegi onal Tri
alCour t.
Howev er,itist obenot edt hatsi nceDeci sionOr der soft heNWRCar eassai led,t hel at t
er
agencyshoul dbei mpl eadedasani ndi spensablepar t ydef endanti nor dert hatanyj udgmentcoul d
beef fectiveandbi ndi ngoni t,andsot hatcompl eterel i
efmaybeaccor dedt ot hepar ti
es.

AMI
STOSOVSONGANDNERI
G.R.No.L-60219
June29, 1984
Ponente:Cuevas

FACTS:Amist
osoandNeriar
eownersofadj
oini
ngpar
celsofagri
cul
turall
and.Anir
ri
gati
oncanal
tr
aver
sesthelandofNer
ithroughwhi
chir
ri
gati
onwaterfr
om theSilmodRiverpassesandand

ATTY.VI
LLEGAS 28
I
NTERNATI
ONALENVI
RONMENTALREGULATI
ONS
ANDNATURALRESOURCESDI
GESTS

fl
owst ot hel andoft heAmi stosof ort helatter'
sbenef icialuse.Ami stosof iledacompl aintf or
Recogni tionofBasementwi thPr eliminar yInjunctionandDamagesagai nstNer iandOng( cultivator
ofNer i
’sl and)f orrefusal ,despi terepeat eddemands, tor ecogni zet her ightsandt itl
eoft hef or mer
tothebenef icialuseoft hewat erpassi ngthrought heirr
igat ioncanal andt ohav eAmi st oso' sr i
ght s
and/ orcl ai msannot atedont heCer tificateofTi tl
eofNer i.Ner ideni edanyr ightofAmi stosoov er
theuseoft hecanal ,norwast hereanycont ract,deedorencumbr anceont hei rpr opertyandasser t
thatt heyhav enotper formedanyactpr ej
udici
alt othepet it
ionert hatwi llwar rantt hef i
lingoft he
compl aintagai nstthem.
Ner iasser t
st hatt hecompl ai ntshoul dbedi smi ssedbecauseAmi stoso’ scl aimi sbased
onhi sr ightt ousewat ercomi ngf rom t heSi l
modRi v erandpr ay st hatAmi stoso’sr i
ghtt ot he
uti
li
zat iont her eofber espect edandnotbedi sturbedand/ orobst ruct edbyNer i
.Thedi sput eist hus
ont heuse, conser vati
onandpr otectionoft her ighttowat erandt heannot ationi smer elyt her eli
ef
prayedf oront hebasi soft hecl aimt ot heuseandpr otect ionofwat erpassi ngt hrought hel andof
Neri.Andsi ncet hecont rov ersyhi ngesont her i
ghtt ouseandpr ot ectt hewat erf r
om t heSi l
mod
Rivert hatpassesont hel andofNer itoAmi stoso'spr oper ty,thepr operaut horityt odet ermi nesuch
acont rov ersyi stheNat ionalWat erResour cesCounci l,whi chisv estedwi thexcl usivej ur i
sdi ction
oversuchquest ion.Thet ri
al cour tdismi ssedAmi stoso’scompl aintf orlackofj ur i
sdiction.

I
SSUE:
1. W/
NAmist
osohastheri
ghtov
ertheuseofthecanal.–YES.
2. W/
NNat
ional
WaterResour
cesCounci
lhasexcl
usivejur
isdi
cti
onov
ert
hemat
ter
.–NO.

RATI O:Basedf r
om t hest i
pul ationoff actsbet weent hepar ti
es, Ner iadmi tst hatAmi stoso, hasan
approv edWat erRi ghtsGr anti ssuedbyt heDepar tmentofPubl icWor ks,Tr anspor t
ati
onand
Communi cati
ons.Ner icont endst hatt hesai dgr antdoesnotper t
aint ot hebenef ici
aluseof
i
rri
gat i
onwat erf r
om Si lmodRi v er.Ther ecor ds,howev er,donotshow anyot heri rrigat i
onwat er
goingt opet it
ioner '
spr opertypassi ngt hrurespondent s'lotasi def rom thatcomi ngf rom t heSi l
mod
River,maki ngNer i’
sallegationsi nvali
d.
Ther ecordcl earlydi scl osesanappr ov edWat erRi ght sGr antinf avorofAmi stoso.The
grantwasmadet hree( 3)y ear sbef oret hepr omul gat i
on ofP. D.1067 ( Wat erCodeoft he
Phili
ppines) .The wat err ights gr antpar takes t he nat ur
e ofa documentknown as a  wat er
permi t
 recogni zedunderAr ti
cle13ofP. D.1067.t heWATERRI GHTSGRANTofAmi stosodoesnot
fal
lunder" claimsf orar ightt ousewat erexi st i
ngonorbef or eDecember31,1974"whi chunder
P.D.1067ar er equiredt ober egi steredwi tht heNat i
onalWat erResour cesCounci lwi thintwo( 2)
yearsf rom pr omul gationofP. D.1067, otherwi sei tisdeemedwai vedandt heuset her eofdeemed
abandoned.
Thegr antcont radictst heer r
oneousf indi ngsoft herespondentJudge, andi ncont r
ov erti
bl y
entit
lespet iti
onert ot hebenef i
ci aluseofwat erf rom SilmodRi ver.Thatr ighti snowa.v estedone
andmaynol ongerbel it
igatedast obr i
ngpet iti
oner 'scasewi t
hint hej urisdictionoft heNat ional
Wat erResour cesCounci l.Tor esurrectt hati ssuewi llbev iolativeoft her uleon  resj udicat a.
Ami stosoi snotaski ngt hecour tt ogr anthimt her i
ghtt ousebutt ocompelNer itor ecogni zet hat
ri
ghtandhav et hesameannot at edont hel atter ’
sTCT.Thei nterrupt i
onoft hef reef l
owofwat er
causedbyt her efusaltor e-opent hecl osedi rrigat i
oncanalconst i
tutedpet i
tioner '
scauseofact ion
i
nt hecour tbel ow,whi chdeci dedl ydonotf allwi thi
nt hedomai noft heaut hor it
yoft heNat ional
Wat erResour cesCounci l
.

LONEYetal
.VSPEOPLE
G.R.No.152644      
Februar
y10,2006
Ponente:Car
pio

FACTS:Loneyetal.
,areoff
icersofMarcopperMiningCor
por
ation(
"Marcopper"
),acor
porat
ion
engaged i
n mini
ng inthe provi
nce ofMarinduque.I
n one ofMarcopper’
s oper
ati
ons,the

ATTY.VI
LLEGAS 29
I
NTERNATI
ONALENVI
RONMENTALREGULATI
ONS
ANDNATURALRESOURCESDI
GESTS

cor porati
ondi schargedmi ll
ionsoft onsoft ai
li
ngs( mi newast e)intotheBoacandMakal upni
t
ri
ver s.TheDOJsepar atel
ychar gedLoneyetal .forviolati
ngv ari
ouslaws( TheWat erCode,The
Mi ningact ,RPC,andt heNat i
onalPollut
ionCont r
olDecr ee).Loneyetal .mov edt oquasht he
i
nf ormat i
onscl aimingthatt heinformati
onswer e"duplicit
ous"ast heDOJchar gedmor ethanone
offensef orasi ngleact.TheMTChel dthatLoneyetal.isliableundert hePhil
ippi
neMi ningActbut
dismi ssedt hev i
olati
onofot herlaws.Onpet i
ti
ont otheRTC,i taffir
medt hedecisi
onandor dered
theot herchar gesr ei
nstated.TheRTCsai dthattherecanbenoabsor pti
onbyoneof fenseoft he
threeot herof fenses,as[ the]act spenali
zedbyt hesel awsar esepar ateanddi st
inctf r
om each
other .Thedi ff
erentlawsi nvolvecannotabsorboneanot herast heelementsofeachcr imear e
differentfr
om oneanot her.Onpet i
ti
ontotheCA, i
taffi
rmedt hedeci si
onoftheRTC.

I
SSUE:
1. W/Nal lt
hechargesf i
ledagai
nstpeti
ti
oner
sexceptoneshoul
dbequashedf ordupli
cit
yof
chargesandonl ythechargef orReckl
essImpr
udenceResult
inginDamaget oPr opert
y
shouldstand.–NO.
2. W/NBr anch94’srul
ing,asaff
ir
medbyt heCour
tofAppeal
s,contr
avenesPeopl
ev .Rel
ova.

RATI
O:
NoDuplici
tyofCharges
Ther
eisduplici
ty(ormulti
pli
cit
y)ofchargeswhenasi ngl
eInformati
onchar gesmor ethan
oneof f
ense.UnderSect i
on3(e),Rule117 ofthe1985Rul esofCr i
minalPr ocedure,dupli
cit
yof
off
ensesi nasingl
einformati
onisagr oundtoquasht heInformati
on.TheRul esprohibi
tthefil
ing
ofsuchInfor
mat i
ontoav oi
dconfusingtheaccusedinpreparinghisdefense. 
Inthiscase,howev er
,
theprosecuti
onchargedeachpet iti
onerwithfouroff
enses,wi theachI nf
ormat i
onchar gi
ngonl y
oneoffense.Thus,Loneyetal.erroneousl
yinvokedupli
cityofchargesasagr oundtoquasht he
I
nformations

TheFi li
ngofSev er
al ChargesisProper
Thef il
i
ngoft hemul ti
plechar gesagainstpetit
ioners,al
thoughbasedont hesamei ncident
,
i
sconsi stentwi thset tl
eddoct ri
net hatwheret wodi f
ferentlaws( orart
iclesoft hesamecode)
defi
net wocr imes,pr iorjeopardyast ooneoft hem i snoobst acletoapr osecuti
onoft heot her
,
al
thoughbot hoffensesar isefrom thesamef acts,ifeachcr i
mei nvol
vessomei mpor tantactwhi ch
i
snotanessent i
alelementoft heot her.Acompar ati
veanalysisbetweent hel awsshowst hateach
oftheselawsonwhi chLoneyetal .wer echarged,thereisoneessent ialel
ementnotr equi
redoft he
others.Mor eover,theof fensespuni shedbyspeci allawar emal aprohibi
tai ncont rastwitht hose
punishedbyt heRev i
sedPenalCodewhi charemal ainse.Thechar geforviolati
onofRPCdoesnot
absorbthechar gesf ortheot herlawsbecausemal ainsef el
oniescannotabsor bmal apr ohibi
ta
cri
mes.

Peoplev .RelovanotinPoi nt
Int hecaseofPeopl ev .Relova,thecour thel
dt hataper sonchargedwi t
ht heftofelectri
c
powerundert he RPC af terbei ng acqui tt
ed ofv i
olating a Ci ty Ordi
nance penal i
zing the
unaut horizedi nstal
lationofel ectri
calwi r
ingv i
olatest
her ightagai nstdoublej
eopar dybecauset he
actgi vi
ngr iset othechar geswaspuni shedbyanor dinanceandanat i
onalstatute,thusfall
ing
withint hepr oscri
ptionagainstmul tipleprosecutionsfort hesameact .However,suchi snott he
caseher ebecauseLoneyetal .arebeingpr osecutedforanactori ncidentpuni
shedbyf ournati
onal
statutesandnotbyanor dinanceandanat ionalstatute.AlthoughLoneyetal .cannotbef or
mul ti
plepr osecut edf orthesameof fense,theycanbechar gedf orof f
ensesari
singf rom thesame
i
ncident .
THEALEXANDRI ACONDOMI NIUM CORPORATI ON(TACC)VS.LAGUNALAKEDEVELOPMENT
AUTHORI TY(LLDA)
G.R.No.169228
Sept ember11, 2009
Ponent e:Car pi
o

FACTS:Phi
l
ippi
neReal
tyandHol
dings,I
nc.(
Phi
l
Real
ty)
,whi
chconst
ruct
edanddev
elopedThe

ATTY.VI
LLEGAS 30
I
NTERNATI
ONALENVI
RONMENTALREGULATI
ONS
ANDNATURALRESOURCESDI
GESTS

AlexandraCondomi nium Compl ex, t


ransferredt oTheAl exandr aCondomi nium Corpor ati
on( TACC)
suchcondomi nium compl exbyv irt
ueofaDeedofConv eyance.Subsequent ly( after5y ears),
LagunaLakeDev elopmentAut hority(LLDA)adv i
sed TACC t hati t
swast ewaterdi d notmeet
governmentef fluentstandards, andi nf
ormedTACCt hati tmustputupi tsownSewageTr eatment
Plant(STP)f ori t
sef fl
uentdischar get omeetgov ernmentst andards.Si nceconst ruct i
nganSTP
would be expensi ve( P15M) ,TACC t ri
ed t o exper i
mentwi t
h ot hermet hods ofcl eani ng i ts
wastewater.Howev er,thewast ewat ersti
llfail
edt omeetgov ernmentst andards.Fort hisv iolation,
LLDA imposedaP1000dai lyf ineonTACC unt i
lt hewast ewat erdi schargecompl ieswi tht he
governmentst andar d.
TACCt henent eredintoanagr eementwi thWor ldChem Mar keti
ngf ortheconst ruct i
onof
theSTPf or 
P7. 5M.LLDAi ssuedanOr derrequi r
ingTACCt opayt hef ine(~P1M)r epresent ingt he
penaltyfr
om unt i
ltheSTPwasconst r
ucted.TACCr equest edLLDAt ocondonet hei mposi tionof
thepenaltyof  
P1,000perdayi nr ecogni t
ionoft her emedi alandcor rectivemeasur esi tunder took
to compl y with gov ernmentst andards.TACC f urtherar gues thatt he non-compl i
ance wi th
governmentst andar dswasduet otheomi ssionandf aultofPhi l
Realty .Thiswasdeni edbyLLDA.
TACCt henf il
edapet iti
onf orcer ti
oraribeforet heCour tofAppeal s(CA)wi thapr ay erf or
theissuanceofat empor aryrestrai
ningor der.TheCAdeni edTACC’ spet it
ion.

I
SSUE:
1. W/
NTACC’
compli
edwiththedoct
ri
neofexhaust
ionofadmi
nist
rat
iver
emedi
es.–NO.
2. W/
NTACCi
stheonel
iabletopayt
hefi
ne.–YES.

RATI
O:
Non- Exhaust ionofAdmi nistr
ativeRemedi es
Thedoct r
ineofnon- exhaust ionofadmi nistrat
iver emediesr equirest hatr esortbef ir
st
madewi t
ht headmi nistr
ativeaut hor it
iesi nt her esoluti
onofacont r
ov ersyf all
i
ngundert hei
r
11
j
ur i
sdictionbef oret hecont r
ov ersymaybeel evat edt oacour tofjusticef orr evi
ew.  
Apr emature
i
nv ocat i
on of a cour t’
si nter vention r ender st he compl aintwi thoutcause of act ion and
12
dismi ssible. I
nt hiscase,TACChasanadmi nistrativ
er ecoursebef oret heDENRSecr et
ar ywhich
i
tshoul dhav ef irstpur suedbef oref i
lingapet iti
onf orcerti
oraribeforet heCour tofAppeal s.Thisis
becauseunderExecut iveorderNo.149t ransf erredLLDAf rom theOf fi
ceoft hePr esi
dentt othe
DENR" forpol icyandpr ogram coor dinat ionand/ oradmi ni
strati
vesuper vi
sion.Andal t
houghunder
thesameor der ,DENRonl yhasadmi nistrativepowerov erLLDA,asubsequentEO192mandat es
theDENRt o" pr omul gat erulesandr egul ationsf ort hecont r
olofwat er,airandl andpol l
ution"and
to" promul gat eambi entandef f
luentst andar dsf orwat erandai rqual i
tyi ncludingt heal l
owable
l
ev elsofot herpol lutantsandr adiations. "

Power softheLLDAt oImposePenal t


y
LLDA, byv ir
tueofitsspecialchart
er,hast her esponsibil
i
tytopr otecttheinhabitantsofthe
LagunaLaker egionf r
om thedeleteri
ousef f
ectsofpol l
utantsemanat i
ngf rom thedi schargeof
wastesf r
om t hesur roundi
ngareas.UnderSect i
on4- AofRA48501,asamended,LLDAi sent i
tl
ed
tocompensat i
onf ordamagesr esulti
ngfrom f ai
l
ur etomeetest abli
shedwat erandef f
luentqualit
y
standards.
Itiscl earthatt heresponsibi
li
tytocompl ywi t
hgov er
nmentst andardsl i
eswi t
hTACC,
becausePhi lRealt
yt urnedoverthepr oj
ecttoTACCf ivey earsbeforeLLDAadv i
sedTACCt hatit
s
wastewat erdid notmeetgov ernmentef fl
uentst andards. I f
,ascl ai
med byTACC,t henon-
compl i
ancewasduet otheomissionandf aultofPhi l
Realty,TACC’ srecoursei stofil
eanact i
on,if
warranted,againstPhi l
Realt
yinapr opercourt.TACCcannotescapei tsli
abil
itytoLLDAbyshi f
ti
ng
theblamet oPhi l
Real t
y.Hence,theLLDAdi dnotabusei tsdiscreti
oni nissuingits4Sept ember
2003Or der.

1Sec.4- A.Compensat i
onf ordamagest othewat erandaquat icresourcesofLagunadeBayandi ts
tr
ibut
ariesresul
ti
ngf r
om fai
luret
omeetest abl
i
shedwat erandeff
luentqual
it
ystandardsandfr
om suchot
her
wrongfulactoromissionofaperson,
priv
ateorpublic,j
uri
dical
orotherwi
se,puni
shableundert
hel
awshallbe
awardedt otheAuthori
tytobeearmar
kedf orwaterqual
itycont
rol
andmanagement .

ATTY.VI
LLEGAS 31
I
NTERNATI
ONALENVI
RONMENTALREGULATI
ONS
ANDNATURALRESOURCESDI
GESTS

LAGUNALAKEDEVELOPMENTAUTHORI
TY(
LLDA)VSCOURTOFAPPEALS(
CA)
G.R.Nos.120865-71
December7, 1995
Ponente:Hermosi
sima,
Jr.

FACTS:RA 4850,whi
chcreatedtheLagunaLakeDev el
opmentAuthor
ity(LLDA),wasparti
all
y
amendedbyMarcosduetotherapi
dexpansi
onofMetroManil
aandit
simpactont heenv
ironment.
Thi
sfurt
herdefi
nedandenl
argedthefuncti
onsandpowersofLLDA.Subsequent
ly,t
heLGCt ook

ATTY.VI
LLEGAS 32
I
NTERNATI
ONALENVI
RONMENTALREGULATI
ONS
ANDNATURALRESOURCESDI
GESTS

effect
,wher et hemuni ci
paliti
esi ntheLagunaLakeRegi oninterpretedt hepr ovi
sionsoft hi
slawt o
meant hatthenewl ypassedl aw gav emuni cipalgov ernment st heexcl usi
v ejuri
sdictiontoi ssue
fi
shingpr ivi
legeswi t
hint heirmuni ci
palwat ers.Muni cipalgov ernment sthereuponassumedt he
authori
tyt oissuefishingpr i
v i
legesandf i
shpenper mi ts.Fi
shpenoper atorstookadv ant ageoft he
occasionwhi chgaverisetoshar pincreaseinunr egulatedfishpenandf i
shcages.
Becauseoft his,t
heLLDAnot i
fiedthepubl i
ct hatallunr egist
er edfishpenorf ishcagesar e
declar
edi ll
egalanddi smant ledot herwisedemol iti
onwi l
lbeef f
ect ed.Theaf f
ectedfishpenowner s
fi
ledinjunctioncasesagai nstt heLLDA.LLDA’ smot i
onst odi smi sswer edeniedbyt heRTCand
affi
rmedbyt heCA.TheCAhel dthatt hepowert ogr antfishingper mi tsisnow v est edwi t
ht he
LGUsandt hepr ov
isionsoft heLLDAchar terinsofarasf ishingpr i
vi
legesi nLagunadeBayar e
concernedhadbeenr epealedbyt heLGC.

I
SSUE/ S:WhichagencyoftheGov ernment(
theLLDAorthetownsandmuni ci
pal
it
iescompr
isi
ng
t
her egion)shoul
dexercisejur
isdicti
onovertheLagunaLakeandi t
senv i
ronsinsof
arasthe
i
ssuanceofpermitsforf
isher
yprivi
legesi
sconcer
ned.–LLDAhasjur
isdi
cti
on.

RATI O:TheLGC donotnecessar il


yr epealt heaf orement ionedl awscr eatingt heLagunaLake
Dev elopmentAut hor i
tyandgr ant i
ngt helat terwat err ightsaut horityov erLagunadeBayandt he
l
aker egion.TheLGCdoesnotcont ainanyexpr esspr ov i
sionwhi chcat egoricallyexpr esslyr epeal
thechar teroft heLLDA.
LLDA’ schar t
erconst it
ut esaspeci allaw whi let heLGCi sagener all aw..I tisbasi ci n
statutoryconst r
uct i
ont hattheenact mentofal aterl egisl
at ionwhi chi sagener allaw cannotbe
const ruedt ohav er epealedaspeci allaw,unl essthei nt entt or epealoral terismani fest,although
thet ermsoft hegener allawar ebr oadenought oi ncludet hecasesembr acedi nt hespeci all aw.
Wher et her eisaconf l
ictbetweenagener allaw andaspeci alstat ut
e,t hespeci alstatuteshoul d
prevailsi ncei tev incest helegi slati
vei nt
entmor ecl ear l
yt hant hegener alst atute.Thus,LLDA’ s
char t
ershoul dpr evailovertheLGC.
Thepoweroft helocalgov ernmentuni tstoi ssuef ishi ngpr ivi
legeswascl ear l
ygr antedf or
revenuepur poses.Ont heot herhand,t hepoweroft heLLDA t ogr antper mi t
sf orf i
shpens,
fi
shcagesand ot heraqua- cul ture structuresi sf ort he pur pose ofef fectivelyr egulati
ng and
moni toringact iv
iti
esi nt heLagunadeBayr egionandf orl akequal i
tycont r
olandmanagement .
Accor dingly,thechar t
eroft heLLDA whi chembodi esav ali
dexer ciseofpol i
cepowershoul d
prevailov ertheLGC onmat tersaf fecti
ngLagunadeBay .Thus,t heLLDA hast heex clusive
j
ur i
sdictiont oi ssueper mitsf ort heenj oy mentoff isherypr ivi
legesi nLagunadeBayt ot he
exclusionofmuni cipal
iti
essituat edt herei
nandt heaut hori
tyt oexer ci
sesuchpower sasar ebyi ts
char t
erv estedoni t
.

TANOv
.SOCRATES
G.R.No.110249
August21,1997
Ponente:Davi
de,
Jr.

FACTS:TANOetal
.fi
l
edapet
it
ionf
orcer
ti
orar
iand 
prohi
bit
ionassai
l
ingt
heconst
it
uti
onal
i
tyof

ATTY.VI
LLEGAS 33
I
NTERNATI
ONALENVI
RONMENTALREGULATI
ONS
ANDNATURALRESOURCESDI
GESTS

Or dinances2 i ssued byt he Sangguni ang Pangl ungsod ofPuer to Pr i


ncesa and i ts Gov er
nor
(Socr ates). Tano et al cont end that the said Or dinances  deprived t
hem  of 
due
process  of
 l
aw,  
their l
iv
elihood,andundul yr est ri
ctedt hem f rom t hepr acticeoft heirt r
ade,in
violationofSect ion2,Ar ti
cleXIIandSect ions2and7ofAr ticl
eXI IIoft he1987Const i
tut i
onand
thatt heMay orhadt he absoluteaut horit
yt odet erminewhet herornott oi ssuet heper mi t.They
alsocl aimt hatitt ookawayt heirri
ghtt oear nt heirli
vel i
hoodi nlawf ulway s;andi nsof arast he
Airline Shipper sAssoci at i
on are concer ned,t heywer e undul ypr evented f rom pur sui ng thei
r
vocat ionandent eri
ng" intocont racts whi ch  arepr oper,necessar y,and  essent ialto carr
y  outthei
r
busi nessendeav orst o asuccessf ulconcl usion.Socr atesetal .def ended t hev ali
dityoft he
Or dinancesasav ali
dexer ci
seoft hePr ov incialGov er nment 's powerundert he general welf
are
clause;  t
hey l i
kewi se mai ntai
ned thatt here was  no v iolati
on oft he due pr ocess and equal
prot ection clauses oft he Const it
ut i
on.Socr ates wtal .reasoned t hatpubl ic hearings wer e
conduct edbef oret heenact mentoft heOr dinancewhi ch,undoubt edl y
,hadal awf ulpurposeand
empl oyedr easonabl emeans;whi l
east ot hel atter,asubst ant i
aldi sti
nctionexi sted" bet weena
fi
sher manwhocat chesl ivefishwi tht hei nt entionofsel l
i
ngi tlive,andaf isher manwhocat ches
l
ivef i
shwi t
hnoi ntenti
onatal lofsel l
ingi tl ive,"i
 e.
. ,"thef ormerusessodi um cy anidewhi lethe
l
at terdoesnot ."Further,theOr dinanceappl iedequal l
ytoal lt
hosebel ongingt oonecl ass.

I
SSUE:
W/Nt
heOr
dinancesi
nquest
ionar
eunconst
it
uti
onal
.–NO.

RATI O:Ther ei sabsol utelynoshowi ngt hatanyoft heTanoetal .qual i


fiesasasubsi stenceor
mar gi nalf isher mant oinv okeasdef enseofv i
ol ationofSect ions2and7ofAr ti
cl eXI I
I.Sect ion7,
particul ar l
y,speaksnotonl yoft heuseofcommunalmar i
neandf i
shi ngr esour ces,butoft heir
protect ion,dev elopmentandconser vation.Asher eaf t
ershown,t heor di nancesi nquest i
onar e
meantpr eciselyt opr otectandconser veourmar i
ner esour cest ot heendt hatt hei renj oymentmay
beguar ant eednotonl yfort hepr esentgener ation, butal sof orthegener ationst ocome.I Naddi t
ion,
Sect ion2ofAr ti
cleXI Iaimspr i
mar i
lynott obest owanyr ightt osubsi stencef isher men,butt ol ay
stressont hedut yoft heSt atet opr otectt henat ion' smar ineweal th.Whatt hepr ov i
sionmer ely
recogni zesi st hatt heSt atemayal low, byl aw, cooper ati
vef ishf armi ng, wi t
hpr iori
t ytosubsi stence
fi
sher menandf ishwor kersi nr iv
er s,lakes, bay sandl agoons.
I
nl ightt henoft hepr inciplesofdecent ralizat ionanddev olutionenshr inedi nt heLGCand
thepower sgr ant edt hereint oLGU underSect ion16( theGener alWel far eCl ause) ,andot her
prov isions,t hev alidityoft hequest i
onedOr di nancescannotbedoubt ed.I ti scl eart hatbot h
Ordi nanceshav et wopr incipalobj ectivesorpur poses:( 1)t oest abl i
sha" closedseason"f orthe
speci esoff ishor  
aquat i
cani malscov eredt her einf oraper i
odoff i
v ey ear s;and( 2)t opr otectthe
corali nt hemar i
newat ersoft he Ci t
yofPuer toPr incesaandt hePr ov i
nceof  Palawanf rom f urt
her
dest ruct ionduet oi l
legal fi
shi ng act i
v i
ties.Fur ther mor e,Or dinancesbanni ngt hecat chingofcer tain
speci esoff ishesandcor alsneednotbeappr ov edbyt heDENRbef oret heycanbeef fective
becausei ntheexer ciseofdev ol
vedpower ,suchappr ov alisnotnecessar y .
TheLGCv est smuni ci
palitieswi t
ht hepowert ograntf i
sher ypr ivil
egesi nmuni cipalwat er s
andi mposer ent als,f eesorchar gest heref or;t openal ize,byappr opr iateor dinances,t heuseof
expl osiv es, noxiousorpoi sonoussubst ances, elect ricity,mur o-ami , andot herdel eteriousmet hods
off ishing;andt opr osecut eanyv iol
at ionoft hepr ov i
sionsofappl i
cabl ef isher yl aws.Fur t
her,the
sangguni angbay an,t hesangguni angpanl ungsodandt hesangguni angpanl alawi ganar edi r
ected
toenactor dinancesf ort hegener alwel far eoft hemuni cipalityandi tsi nhabi tants,whi chshal l
i
ncl ude,i nteral i
a,or dinancest hat" [p]rotectt heenv ironmentandi mposeappr opr i
atepenal t
iesf or

2( 1)
 OrdinanceNo.  15-
92" ANORDI NANCEBANNI NGTHESHI PMENTOFALLLI VEFISHANDLOBSTER
OUTSI DE PUERTO PRI NCESA CI TY FROM JANUARY 1,1993  TO JANUARY 1, 1998AND PROVI DING
EXEMPTI ONS, PENALTIESANDFOROTHERPURPOSESTHEREOF"
(2)Of fi
ceOr derNo.23,  requiring any 
person engaged or 
i
ntending t
o  engagei nany busi
ness,tr
ade,
occupation,cal l
i
ngorpr of essionorhav i
ngi nhispossession anyoft hear ti
clesforwhichaper miti
s
requi
redt obehad, toobtainf i
rstaMay or’
sandaut horizi
nganddi r
ect
ingt ocheckorconductnecessary
i
nspect i
onsoncar goescont aininglivefi
shandl obsterbei
ngshippedoutf r
om PuertoPri
ncesaand,
(3) 
Resolution No. 
33, 
Ordinance  No. 
2 enti
tl
ed: 
"A RESOLUTION PROHIBI TING THECATCHI NG, 
GATHERIN
G, 
POSSESSI NG, BUYING, 
SELLI NG  AND SHIPMENT  OF LIVE 
MARI NECORAL DWELLING AQUATIC
ORGANI SMS”

ATTY.VI
LLEGAS 34
I
NTERNATI
ONALENVI
RONMENTALREGULATI
ONS
ANDNATURALRESOURCESDI
GESTS

actswhi chendangert heenv i


ronmentsuchasdy namitefi
shingandotherf ormsofdest ruct
ive
fi
shingandsuchot heractivi
ti
eswhi chresultinpol l
uti
on,acceler
ati
onofeut r
ophicati
onofr i
vers
andlakes,orofecologicali
mbal ance.Oneoft hedev olv
edpower senumerat
edi nthesecti
onoft he
LGC on dev oluti
on ist he enf or
cementoff isherylaws in munici
palwat ersi ncl
udi
ng t he
conservati
onofmangr oves.Thisnecessari
lyi ncludestheenactmentofordinancest oeff
ectiv
ely
carr
youtsuchf isheryl
awswi thinthemunicipal waters.

HI
ZONetal
.VSCA
G.R.No.119619
December13,1996
Ponente:Puno

FACTS:HI
ZONetal.werechar
gedwithvi
olat
ingPD704forsupposedl
yfi
shi
ngwit
houtt
heuseofa
poi
sonoussubst
ance(sodi
um cyani
de)
.Ar eportt
hatsomef i
shi
ngboatswerefi
shi
ngby"muro

ATTY.VI
LLEGAS 35
I
NTERNATI
ONALENVI
RONMENTALREGULATI
ONS
ANDNATURALRESOURCESDI
GESTS

ami"ledtot heappr ehensionofsuchboat( F/BRobi nson),whereHi zonetalwer epr esent


.The
poli
ce(PNPMar i
ti
meCommandandt heTaskFor ceBant ayDagat)directedtheboatcapt aintoget
random samplesoft hefishfrom thefishcagef ortest
ing.Thei ni
ti
al r
esultstestedthefishposit
ive
forsodium cyanideandt hatwast hebasisoft heinfor mat
ionagai nstHi zonetal .Howev er
,a
secondsetoff i
shsampl esy i
eldedanegat iv
er esultonthesodi um cyanide.
Notwit
hstanding t hi
s,t he RTC f ound Hi zon etal .guilt
y and sent enced them t o
i
mpr i
sonmentandf orfei
tur
eoft hefishes.TheCAaf fi
rmedt hi
sdeci si
on.Hi zonetal.,t
ogetherwith
theSoli
citorgeneralnowquest i
ont headmi ssibi
li
tyoft heevidenceagai nstpet i
ti
onersinv i
ewof
thewarrantl
esssear chofthef i
shingboatandt hesubsequentar r
estofpet i
tioners.

I
SSUE/S:
1. W/NfishsamplesseizedbytheNBIi ntheF/BRobi
nsonwithoutasearchwarrantar
e
admi
ssibl
einev
idence.–YES.
2. W/NHizonetal
.,ar
eguilt
yofi
l
legal
fishi
ngwit
htheuseofpoi
sonoussubst
ances.NO.

RATI O:Asagener alr ule,anyev idenceobt ainedwi thoutaj udi ci


alwar rantisi nadmi ssibleforany
purposei nanypr oceedi ng.Ther ulei s,howev er,subjectt ocer t
ainexcept i
ons.Sear chandsei zur e
wi t
houtsear chwar rantofv essel sandai rcraftsforv iolationsofcust omsl awshav ebeent he
traditionalexcept iont ot heconst itutionalr equirementofasear chwar r
ant.Thesameex cept i
on
oughtt oappl yt osei zur esoff ishingv essel sandboat sbr eachi ngourf isher ylaws.
Hizonetal .wer echar gedwi thi ll
egalf i
shingpenal i
zedundersect ions33and38ofP. D.
704.Thesepr ov i
si onscr eat eapr esumpt ionofgui ltforpossessi onofexpl osivesorpoi sonous
subst ances.Howev er ,t hispr esumpt ioni smer elypr i
maf aci eandt heaccusedhast her i
ghtt o
presentev idencet or ebutt hispr esumpt ion.
I
nt hiscase, theonl ybasi sf ort hechar geoff ishingwi thpoi sonoussubst ancei sther esult
oft hef ir
stNBIl abor at oryt estont hef ourf i
shspeci mens.Theappr ehendi ngof f
icer swhoboar ded
andsear chedt heboatdi dnotf i
ndanysodi um cy anidenoranypoi sonousorobnoxi oussubst ance.
Neit herdidt heyf indanyt raceoft hepoi soni nt hepossessi onoft hef ishermenori nt hef ishcage
i
tsel f.Undert heci rcumst ancesoft hecase,howev er,thisf indingdoesnotwar rantt hei nfall
ible
concl usiont hatt hef ishesi nt heF/ BRobi nson, orev ent hesamef ourspeci mens, wer ecaughtwi th
theuseofsodi um cy ani de.
Appar ently ,itwast hepol icewhower etheonesengagedi nani l
legalf ishingexpedi ti
on.
"Mur oami ",aswhatwasr epor tedt hef i
sher menwer edoi ng, i
smadewi th"theuseofabi gnetwi th
sinker st omaket henetsubmer gei nt hewat erwi tht hef ishermensur round[ i
ng]t henet ." 
Thi s
met hod off ishing needs appr oxi mat elyt wo hundr ed ( 200)f i
sher men t o execut e. 
Whatt he
appr ehendi ngof ficer si nst eaddi scov eredwer et went yei ght( 28)f isher meni nt hei rsampans
fi
shi ngbyhookandl ine.Theaut hor i
tiesf oundnot hingont heboatt hatwoul dhav ei ndicatedany
form ofi l
l
egalf i
shi ng.Al lthedocument soft heboatandt hef i
sher menwer einor der.I twasonl y
aftert hef i
shspeci menswer et ested,al bei tundersuspi ciousci r
cumst ances,t hatpet i
tionerswer e
char gedwi thill
egal fishi ngwi t
ht heuseofpoi sonoussubst ances.

PROVI
NCEOFRI
ZAL,
etal
.v.EXECUTI
VESECRETARY,
etal
.
GR129546,13
December2005
Ponent
e:Chi
co-Nazar
io

FACTS:AMemor
andum ofAgreementwassi
gnedbyDPWH,DENRandt heMetropol
it
anMani
la
Commissi
on(
MMC)Governor
,all
owingDENRt
outi
li
zeBocaue,
Rizalasasani
tar
ylandf
il
lbyMMC.

ATTY.VI
LLEGAS 36
I
NTERNATI
ONALENVI
RONMENTALREGULATI
ONS
ANDNATURALRESOURCESDI
GESTS

Howev er,theSangguni angBay an( SB)ofSanMat eowr otet oMMC, DPWH, theExecut i
v eSecr etary,
andt heDENR, informi ngt hem oft heSBr esol utionbanni ngcr eati
onofdumpsi tesf orMet roMani l
a
wi t
hini t
sj ur
isdi ction.SBal soaskedt osuspendal loper at i
onswi thr espectt otheSanMat eo
Landf i
llDumpsi te.Noact i
onwast akenont hesel etters.Itwasal sof oundoutt hatt hel andsubj ect
oft heMOAwaspar toft heMar ikinaWat er shedReser vationAr ea.Thus,t her epor tsubmi ttedby
thef orestof fi
cer soft heFor estEngi neer i
ngandI nf rastruct ur eUni toft heCommuni tyEnv i
ronment
andNat ur alResour ceOf fice( CENRO) ,DENR- IV,Ri zalPr ov i
nce,r ev eal edt hatt her ewasnoper mi t
i
ssuedt oMMCt out ili
zet hesepor tionsofl andf ordumpi ngpur poses.I tfurt herst atest hatt heuse
oft hear easasdumpi ngsi tegr eat l
yaf fectst heecol ogicalbal anceandenv i
ronment alf actor sin
thatcommuni t
y .
DENR subsequent l
y gr ant ed t he Met ropol itan Mani la Aut hor i
ty( for mer ly MMC)an
Env i
r onment alCompl i
anceCer ti
f i
cat e( ECC)f ort heoper ationoft hegar bagedumpsi te.Lesst han
sixmont hsaf tert hei ssuanceoft heECC,DENRsuspendedt heECCi nal etteraddr essedt ot he
DPWH,st ati
ngt hati twasascer tai nedt hatgr oundsl umpi ngander osionhav er esul t
edf r
om
i
mpr operdev elopmentoft hesi te.DENRt hensental et tert oMMAr ecommendi ngt hatt heal l
facili
tiesandi nf r
ast ruct urei nt hegar bagedumpsi t
ei nBocauebedi smant led.Despi tet hev ar i
ous
object ionsandr ecommendat ionsr ai sedbyt hegov ernmentagenci es,t heOf ficeoft hePr esi dent,
throughExecut i
v eSecr et aryRubenTor res,si gnedandi ssuedPr ocl amat i
onNo.635,“ Excl uding
from t heMar ikinaWat ershedReser v ationCer tainPar celsofLandEmbr acedTher ei nf orUseas
Sani taryLandf i
llSi tesandSi mi larWast eDi sposalUndert heAdmi ni strati
onoft heMet ropol i
tan
Mani l
aDev el
opmentAut hor ity
.”
Pr ovinceofRi zal (
PROVI NCE)f i
ledbef oret heCAaci vilactionf orcer t
ior ar
i,pr ohi biti
onand
mandamuswi thappl icat i
onf orat empor aryr est rainingor der/wr i
tofpr elimi nar yi njunct ion.CA
deni edt hepet iti
onf orl ackofcauseofact i
on.Whi let heappealwaspendi ng,Pr ov i
ncef ileda
Mot ionf orTempor aryRest rai
ni ngOr der ,poi ntingoutt hatt heef fect soft heElNi ñophenomenon
woul dbeaggr av atedbyt her elent lessdest ructionoft heMar i
kinaWat ershedReser vation.Asa
resul t,
MMDAof f
icialsagr eedt oabandont hedumpsi teaf tersi xmont hs
Pr esidentJosephE.Est r adai ssuedaMemor andum or der i
ngt hecl osur eoft hedumpsi t
e
anddi rect edDI LG Secr etaryAl fredoLi m andMMDAChai rmanBi nayt or eopent heSanMat eo
dumpsi t
e“ i
nv iewoft heemer gencysi tuat i
onofuncol lect edgar bagei nMet r oMani l
a, resul ti
ngi na
criti
calandi mmi nentheal t
handsani tationepi demi c.”Cl ai mingt heabov eev ent sconst it
ut eda
“clearandpr esentdangerofv iol enceer upt ingi nt heaf fect edar eas, ”t hePr ov incef iledanUr gent
Pet i
tionf orRest rainingOr der.TheSCi ssuedt heTRO.Meanwhi le,RA.9003,ot herwi seknownas
“TheEcol ogical Sol idWast eManagementActof2000, ”wassi gnedi ntol awbyPr esidentEst rada.

I
SSUE/S:
1. W/Nr espondentMMDAagr eedt
otheper
manentcl
osur
eoftheSanMateoLandfil
lasof
December2000; and
2. W/Ntheper manentclosur
eoftheSanMat
eol
andfi
ll
ismandat
edbyRep.ActNo.9003.

RATI O:SanMat eoLandf il


lwi l
lremai npermanent l
yclosed.Pr oclamat i
onNo.635i si l
l
egal.The
Cour theldt hatamer eMOAdoesnotguar anteet hedumpsi te’
sper manentcl osur e.TheSanMat eo
sitehasadv erselyaffectedi t
senv i
rons,andsour cesofwat ershoul dalway sbepr ot ect
ed.Adv erse
effectsoft hesi tewer erepor t
edasear l
yasofJune1989.MMAwasal soi nfor medt hattheheav y
polluti
onandr i
skofdi seasegener atedbydumpsi t
esr enderedt helocationofadumpsi tewithin
theMar ikinaWat ershed Reser vati
on incompat iblewi t
h LagunaLakeDev el opmentAut hority’
s
(LLDA)pr ogr am ofupgr adingt hewat erquali
tyoft heLagunaLake.I nvesti
gat ionRepor tsregarding
ther espi
r ator yil
lnessesamongpupi l
sofapr imar yschoollocatedappr oximat ely100met ersfrom
thesi t
e,aswel last heconst antpresenceofl argef li
esandwi ndblowndebr i
sal lov ertheschool ’
s
playgroundwer eal sosubmi t
ted.Leachatetreatmentpl anterodedt wicealready ,cont aminati
ngt he
near bycreekst hatwer esour cesofpot ablewat erforther esidents.Thecont ami natedwat erwas
alsof oundt of l
owt otheWawaDam andBoso- bosoRi ver,whi chinturnempt i
esi ntoLagunade
Bay .
Pr otectionofwat ershedsisan“ int
ergener ati
onal”responsibil
it
ythatneedst obeanswer ed
now.Thr eeshor tmont hsbef or
ePr oclamationNo.635waspassedt oav ertt hegar bagecr isis,
Congr esshadenact edt heNat i
onalWat erCr i
si
sActt o“ adoptur gentandef fectivemeasur est o

ATTY.VI
LLEGAS 37
I
NTERNATI
ONALENVI
RONMENTALREGULATI
ONS
ANDNATURALRESOURCESDI
GESTS

addr esst henat ionwi dewat ercr isiswhi chadv er sel yaf fect st heheal t
handwel l-beingoft he
popul at ion,f oodpr oduct i
on,andi ndust r
ial i
zat ionpr ocess.Oneoft hei ssuest hel aw soughtt o
addr esswast he“ prot ect ionandconser vat ionofwat ersheds. ”Respondent s’ act ionsi nt hef aceof
suchgr av eenv ir
onment alconsequencesdef yal llogi c.Thepet i
t i
oner sr ightlynot edt hati nst eadof
prov idi ngsol utions, theyhav e, wi thunmi t
igat edcal lousness, wor senedt hepr obl em.
TheReor gani zat ionActoft heDENR def inesandl imi tsi tspower sov ert hecount r
y ’
s
nat ur alr esour ces.TheAdmi nist rati
veCodeof1987andExecut iv eOr derNo.192ent rustt heDENR
wit ht heguar dianshi pandsaf ekeepi ngoft heMar ikinaWat er shedReser v ationandourot her
nat ur alt reasur es.Al thought heDENR ownst heMar ikinaReser ve,i tisbuti sdef i
nedbyt he
decl ar edpol iciesoft hest at e,andi ssubj ectt ot hel aw andhi gheraut hor i
t y.Sect ion2,Ti tleXI V,
BookI Voft heAdmi ni strat iveCodeof1987, whi l
especi fi
cal lyr ef erringt othemandat eoft heDENR,
makespar ti
cul arr efer encet ot heagency ’
sbei ngsubj ectt ol aw andhi gheraut hor i
t y.Wi t
hgr eat
powercomesgr eatr esponsi bilit
y.I ti st hehei ghtofi ronyt hatt hepubl icr espondent shav e
vigor ousl yar rogat edt ot hemsel v esthepowert ocont rol theSanMat eosi t
e, buthav edef tl
yi gnor ed
theircor respondi ngr esponsi bi l
ityasguar di ansandpr ot ect or soft hist or ment edpi eceofl and.
TheLocalGov er nmentCodegi v est oLGUsal lt henecessar ypower st opr omot et he
gener alwel far eoft hei ri nhabi tant s.Theci rcumst ancesunderwhi chPr ocl amat i
onNo.635was
passedal sov i
ol atesR. A.No7160,ort heLocalGov er nmentCode.Pr oclamat ionNo.635,whi ch
waspassedon28August1995,i ssubj ectt ot hepr ov i
sionsoft heLocalGov er nmentCode,whi ch
wasappr ov edf oury ear sear li
er , on10Oct ober1991.Sect ion2( c)oft hesai dl awdecl ar est hati tis
the pol icy oft he st ate“ tor equire al lnat ionalagenci es and of fices t o conductper i
odic
consul tations wi th appr opr iate l ocal gov er nment uni t s, non- gov ernment al and peopl e's
organi zat i
ons,andot herconcer nedsect or soft hecommuni tybef or eanypr ojectorpr ogr am i s
i
mpl ement edi nt hei rr espect ivej urisdi ctions. ”Li kewi se,Sect i
on27r equi respr i
orconsul tati
ons
bef or eapr ogr am shal lbei mpl ement edbygov er nmentaut hor itiesandt hepr iorappr ov aloft he
sangguni ani sobt ai ned.
Undert heLocalGov ernmentCode, theref or e, twor equi sitesmustbemetbef oreanat i
onal
proj ectt hataf fect st he env ironment aland ecol ogi calbal ance ofl ocalcommuni t
ies can be
i
mpl ement ed:pr iorconsul tationwi tht heaf f
ect edl ocalcommuni t i
es,andpr iorappr ov aloft he
proj ectbyt he appr opr iat e sangguni an.Absentei t heroft hese mandat or yr equi rement s,t he
proj ect ’si mpl ement at ioni si llegal .
Appr ov edon26Januar y2001, “TheEcol ogi calSol i
dWast eManagementActof2000”was
enact edpur suantt ot hedecl ar edpol icyoft hest at e“ t oadoptasy st emat ic,compr ehensi v eand
ecol ogi calsol idwast emanagementsy stem whi chshal lensur et hepr otect i
onofpubl icheal thand
env ironment ,andut i
lizeenv ironment al l
ysoundmet hodst hatmaxi mi zet heut ili
zat ionofv al uable
resour cesandencour ager esour ceconser v at i
onandr ecov ery .”Itr equi rest headher encet oaLocal
Gov er nmentSol idWast eManagementPl anwi thr egar dt ot hecol lect i
onandt ransf er ,pr ocessi ng,
sour cer educt ion,r ecy cling,compost ingandf inaldi sposalofsol i
dwast es,t hehandl i
ngand
disposalofspeci alwast es,educat i
onandpubl ici nf ormat i
on,andt hef undi ngofsol i
dwast e
managementpr oject s.
Thesai dl awmandat est hefor mul at ionofaNat ional Sol idWast eManagementFr amewor k,
whi chshoul di ncl ude,amongot hert hings,t hemet hodandpr ocedur ef ort hephaseoutandt he
ev ent ualcl osur ewi t
hi nei ght eenmont hsf rom ef fect i
v ityoft heActi ncaseofexi st i
ngopendumps
and/ orsani tar yl andf illsl ocat edwi t
hinanaqui fer ,gr oundwat err eser v oirorwat ershedar ea.Any
l
andf i
llssubsequent lydev el oped mustcompl ywi tht hemi ni mum r equi rement sl aid down i n
Sect ion40,speci ficallyt hatt hesi tesel ect edmustbeconsi st entwi tht heov eral llandusepl anof
thel ocal gov er nmentuni t,andt hatt hesi t
emustbel ocat edi nanar eawher et hel andf ill
’soper ation
willnotdet riment allyaf fectenv ir
onment al lysensi tiver esour cessuchasaqui fers,gr oundwat er
reser voi rsorwat er shedar eas.

CI
TYOFBAGUI
OVSMASWENG
G.R.No.180206       
Februar
y4,2009
Ponente:Ti
nga

FACTS:TheBagui
oCi
tyMay
oror
der
eddemol
i
tionofi
l
legalst
ruct
uresl
ocat
edi
nBusolWat
ershed

ATTY.VI
LLEGAS 38
I
NTERNATI
ONALENVI
RONMENTALREGULATI
ONS
ANDNATURALRESOURCESDI
GESTS

Reser vation.Gumangan etal .,opposed t he demol it


ion,cl ai
mi ng thatt he l ands wheret heir
resident i
alhousesst andaret heirancestrallandswhi cht heyhav ebeenoccupy ingandpossessi ng
openl yandcont i
nuousl ysincet i
mei mmemor i
al.Theyf urt
hercl aimt hattheirowner shi
p overthe
l
andshasbeenexpr esslyrecogni zedProclamat ionNo.15dat edApr il27,1922andr ecommended
byt heDepar tmentofEnv i
ronmentandNat ur alResources( DENR)f orexclusionf rom thecoverage
oft heBusol ForestReser v
e.Masweng, Regional HearingOf f
iceroft heNCI PissuedTROst orefrain
from enf orci
ngt heDemol i
tion,andsubsequent l
ytheNCI Pgr antedawr itofpreli
mi nar
yinj
uncti
on.
TheMay orcontendst hatt heNCI Phasnoj ur
isdicti
ont ohearanddeci demai nacti
onsf or
i
nj unctionasi twasnotacasependi ngbef oresuchCommi ssion.Anot hercont enti
onisthatthe
IPRAt hatBaguioCi tyshallbegov ernedbyi t
sChar t
er.Thus, priv
ater espondent scannotcl ai
mt heir
allegedancest rallandsundert hepr ovi
si
onsoft heIPRA.Last l
y,theMay orclai mst hattheBusol
For estReser vationhadal readybeendecl aredbyj uri
sprudenceasi nal
ienableandpossessi on
ther eof,nomat t
erhowl ong, cannotconv ertthesamei ntopr i
vatepr oper t
y.
TheCAuphel dthej uri
sdictionoftheNCI Pandaf f
ir
medt heTROs.

I
SSUE/S:
1. W/NtheNCIPhast hejuri
sdi
cti
onov ert
hemat ter
.–YES.
2. W/NBaguioCit
yisexemptf rom I
PRA.–NO.
3. W/NGumanganetal .

sancest r
allandclai
mi srecogni
zedbybyProcl
amat
ionNo.15,i
n
whi
chcase,t
heirr
ighttheret
omaybepr otectedbyaninj
unct
ivewr
it
.–NO.

RATI
O:
TheNCI Phasj urisdict
ion
I
nor dertodet erminewhet hert heNCI Phasj uri
sdicti
onovert hedi spute,iti
snecessaryt o
resol
ve,ont hebasisoft heal l
egationsi nt heirpeti
tion,whet herpri
v aterespondent saremember s
ofICCs/IPs.Privaterespondent s,asmember soft heI bal
oitri
be,wer easser t
ingownershipov er
porti
onsofBusolFor estReser vat
ionwhi cht heyclaimt obet hei
rancest rallands.Thepet i
ti
onfor
i
njuncti
onsoughtt opr eventtheenf orcementoft hedemol i
ti
onor dersissuedbyt heCityMay or.
Thesethusqual if
iestheact ionasasa" dispute(s)orcont r
oversy(
s)ov erancest r
all
ands/domains
ofICCs/IPs"withi
nt heoriginalandexclusi vejurisdi
ct i
onoft heNCI P.
TheI PRA,fur t
hermor e,endowst heNCI Pwi tht hepowert oissuet emporaryrest
raini
ng
ordersandwr i
tsofi nj
unct i
onwi thoutanypr ohibi
tionagai nstthei ssuanceoft hewr i
twhent he
mainactionisforinjuncti
on.

Bagui
oCityisnotexemptfrom I
PRA.
Al
thoughIPRAstatest hati
sgov ernedbyitsownchar ter,i
tsexempti
onfrom theIPRA,
howev er,cannoti
psofactobededucedbecauset helawconcedest heval
idi
tyofpr
iorlandrights
recognizedoracquir
edthroughanyprocessbef orei
tsef
fect
ivi
ty.TheIPRAdemandst hatthecit
y’s
charterrespectt
hevali
dit
yoftheserecognizedlandri
ght
sandt i
tles.

Procl amat ion15doesnotr ecognizet heancest rallandcl aim


Beforeawr i
tofpr eliminar yi
njunct i
onmaybei ssued,pet i
tioner smustshow t hatt here
existsar i
ghtt obepr ot ectedandt hattheact sagainstwhi chinj uncti
oni sdi rect edar ev i
ol ati
veof
saidr ight.ProclamationNo.15,howev er,doesnotappeart obeadef i
nitiverecogni ti
onofpr ivate
respondent s’ancestr
all andcl aim.Thepr oclamati
onmer elyident i
fi
est heMol intasandGumangan
fami l
ies, t
hepr edecessor s-i
n-interestofpr i
vaterespondent s,ascl aimant sofapor tionoft heBusol
ForestReser vati
onbutdoesnotacknowl edgev estedr i
ght sov erthesame.I nf act,Pr oclamat ion
No.15expl i
citl
ywithdr awst heBusolFor estReservationf rom sal eorset t
lement .Thef actr emains,
too, t
hatt heBusol For estReser v
ationwasdecl ar
edbyt heCour tasi nalienablebyj urisprudence.
Thedecl arat
ionoft heBusolFor estReservationassuchpr ecludesi t sconv ersioni nto
private pr operty.Relat edly ,t he courts ar e notendowed wi thj urisdicti
onalcompet ence t o
adjudicat eforestlands.Al thought heNCI Phast heaut hor i
t ytoi ssuet empor ar yr estraini
ngor ders
andwr i
tsofi njuncti
on,wear enotconv incedthatpr i
v ater espondent sar eent itl
edt ot her eli
ef
grantedbyt heCommi ssi on.

ATTY.VI
LLEGAS 39
I
NTERNATI
ONALENVI
RONMENTALREGULATI
ONS
ANDNATURALRESOURCESDI
GESTS

PROVINCEOFNORTHCOTABATOV.GOVERNMENTOFTHEPHI
LIPPI
NESPEACEPANEL
G.R.No.183591      
October14,2008
Ponente:Carpi
oMorales

FACTS:The Phil
ippi
nes and the MI
LF,t
hrough the Chai
rper
sons oftheirrespect
ive peace
negot
iat
ingpanel
s,wereschedul
edtosi
gnaMemor andum ofAgreementontheAncestr
alDomai n

ATTY.VI
LLEGAS 40
I
NTERNATI
ONALENVI
RONMENTALREGULATI
ONS
ANDNATURALRESOURCESDI
GESTS

(MOA- AD)Aspectoft heGRP- MI LFTr i


poliAgr eementonPeaceof2001i nKual aLumpur ,Mal ay si
a.
Thesi gningoft heMOA- ADbet weent heGRPandt heMI LFdi dnotpusht hroughbecause
uponmot ionofpet i
ti
oner s,speci fical l
ythosewhof iledt hei rcasesbef or etheschedul edsi gningof
theMOA- AD,t hisCour ti ssuedaTempor ar yRest r
ainingOr derenj oini
ngt heGRPf rom si gningt he
same.
TheMOA- ADwaspr ecededbyal ongpr ocessofnegot i
ationandt heconcl udingofsev eral
pri
oragr eement s bet ween t he t wo par ties begi nni ng i n 1996,when t he GRP- MI LF peace
negot i
ati
onsbegan.
TheSol icit
orGener al
,whor epresent srespondent s,summar izest heMOA- ADbyst atingt hatt he
samecont ained,amongot hers,t hecommi t
mentoft hepar t
iest opur suepeacenegot i
at i
ons,
protectandr especthumanr i
ght s,negot i
atewi thsi ncer ityi nt her esolut i
onandpaci fi
cset tlement
oftheconf lict
,andr ef rainf r
om t heuseoft hreatorf or cet oat tainundueadv antagewhi let he
peacenegot iati
onsont hesubst ant iveagendaar eon- goi ng.
OnJul y23,2008,t hePr ov inceofNor t
hCot abat oandVi ce-Gov er norEmmanuelf i
leda
peti
tion forMandamus and Pr ohi biti
on wi t
h Pr ay erf ort he Issuance ofWr i
tofPr eli
mi nar y
Inj
unctionandTempor ar yRest rai ningOr der. I
nv okingt her ightt oinfor mat i
ononmat t
er sofpubl i
c
concern,pet i
ti
onersseekt ocompelr espondent stodi scl oseandf urnisht hem t hecompl et eand
offi
cialcopi esoft heMOA- ADi ncl udi ngi tsat tachment s,andt opr ohibitt hesl atedsi gningoft he
MOA- AD,pendi ngt hedi sclosur eoft hecont entsoft heMOA- AD andt hehol dingofapubl i
c
consultati
on t hereon. Suppl ement aril
y, pet it
ioner s pr ay t hat t he MOA- AD be decl ared
unconst i
tutionalinsof arasbycr eat i
ngandr ecogni zingt heBangsamor oJur i
dicialEnt i
tyasa
separatest ateoraj ur i
di cal,ter ritori
alorpol it
icalsubdi v ision,itwoul dconf li
ctwi thAr ti
cleX,
Section20oft heConst i
t ution,amongot her s.

I
SSUE:W/NtheMOA-AD i
sconsti
tut
ional?NO,ITI S NOT BECAUSE I
TS ADOPTI
ON WOULD
ENTAI
LTHEAMENDMENTOFARTICLE10, SECTION20OFTHECONSTI TUTION,AMONGOTHERS.
THEBJEI
SNOTJUSTANAUTONOMOUSREGI ON,I
TISALMOSTLI KEASTATE.

RATI O:TheMOA- ADi sinconsi stentwi tht heConst i


tutionandl awsaspr esent l
ywor ded.I ngener al
,
theobj ect i
onsagai nstt heMOA- ADcent eront heext entoft hepower sconcededt hereint ot heBJE.
Pet i
ti
oner s asser tt hatt he power s gr ant ed t ot he BJE exceed t hose gr anted t o anyl ocal
gov ernmentunderpr esentl aws,and ev en go bey ond t hose oft he pr esentARMM.Bef or e
assessi ngsomeoft hespeci ficpower st hatwoul dhav ebeenv est edi nt heBJE,howev er ,itwoul d
beusef ult ot urnf i
rstt oagener ali deat hatser vesasauni fyingl inkt ot hedi fferentpr ov isionsof
theMOA- AD, namel y,t heinter nat ional awconceptofassoci
l at i
on.
Fi
r stly,whati sanassoci ation?“ [a]nassoci ationi sf ormedwhent wost atesofunequal
powerv olunt ari
lyest abl i
shdur abl elinks.I nt hebasi cmodel ,onest ate,t heassoci ate,del egates
certainr esponsi bilit
iest ot heot her,t hepr incipal ,whi lemai ntai
ni ngi tsi nternat i
onalst atusasa
state.Fr eeassoci ationsr epr esentami ddl egr oundbet weeni ntegr ati
onandi ndependence. ”
TheMOA- ADi tcont ainsmanypr ovisionswhi char econsi stentwi tht hei nternationall egal
conceptofassoci ation,speci fical l
yt hef ollowi ng:t heBJE' scapaci tyt oent erint oeconomi cand
trader elationswi thf or ei
gncount r
ies,t hecommi tmentoft heCent r
alGov er nmentt oensur et he
BJE' spar tici
pat ioni nmeet i
ngsandev ent si nt heASEANandt hespeci alizedUNagenci es, andt he
cont i
nui ngr esponsi bi li
tyoft heCent ralGov ernmentov erext ernaldef ense.BJEwasal sogr anted
ther i
ghtt opar t
icipat ei nPhi lippi neof ficialmi ssionsbear i
ngonnegot i
at i
onofbor deragr eement s,
env i
ronment alpr otect ion, andshar ingofr ev enuesper tai
ningt ot hebodi esofwat eradjacentt oor
betweent hei slandsf ormi ngpar toft heancest raldomai n.Thesepr ov i
si onsoft heMOAi ndi cate,
amongot hert hings, thatt hePar t i
esai medt ov esti nt heBJEt hest atusofanassoci atedst ateor ,
atanyr ate, ast atuscl oselyappr oximat i
ngi t.
Theconceptofassoci at ioni snotr ecogni zedundert hepr esentConst itution.
Nopr ovince,ci ty
,ormuni cipality,notev ent heARMM,i sr ecogni zedunderourl awsas
hav i
ngan" associ ative"r elationshi pwi tht henat ionalgov ernment .I ndeed,t heconcepti mpl i
es
power st hatgo bey ond any thing ev ergr ant ed byt he Const i
tution t o anyl ocalorr egional
gov ernment .I talsoi mpl iest her ecogni tionoft heassoci at edent ityasast ate.TheConst i
tution,
howev er,doesnotcont empl at eanyst at ei nt hisj urisdict
ionot hert hant hePhi l
ippineSt ate,much
l
essdoesi tpr ov i
def orat r
ansi tor yst atust hatai mst opr epar eanypar tofPhi li
ppi net erritoryf or

ATTY.VI
LLEGAS 41
I
NTERNATI
ONALENVI
RONMENTALREGULATI
ONS
ANDNATURALRESOURCESDI
GESTS

i
ndependence.
Eventhemer econceptani
mat
ingmanyoft heMOA- AD'sprovi
sions,therefor
e,already
requi
resf ori
tsv ali
dit
ytheamendmentofconst
it
utionalpr
ovisi
ons,specifi
call
yt hef ol
l
owi ng
provi
sionsofArt
icleX:

SECTI ON 1.Thet erritori


alandpol iticalsubdi v i
sionsoft heRepubl icoft hePhi l
i
ppi nesar et he
prov inces,ci t
ies,muni cipalities,andbar angay s.Ther eshal lbeaut onomousr egionsi nMusl i
m
Mi ndanaoandt heCor dil
lerasasher einafterpr ov ided.
SECTI ON 15.Ther e shal lbe cr eated aut onomous r egions i n Musl im Mi ndanao and i nt he
Cor dillerasconsi stingofpr ov i
nces,ci ti
es,muni cipaliti
es,andgeogr aphi calar easshar i
ngcommon
anddi st i
nct ivehi stor i
calandcul turalher itage,economi candsoci alst ructures,andot herr el evant
char act eristicswi thint hef ramewor koft hisConst ituti
onandt henat i
onalsov erei
gnt yaswel las
territ
or ialint egr i
tyoft heRepubl icoft hePhi li
ppi nes.
Theseamendment swoul dbenecessar ybecauset heBJEi saf armor epower fulent it
y
thant heaut onomousr egionr ecogni zedi nt heConst itution.Itisnotmer elyanexpandedv er sionof
the ARMM,t he st atusofi tsr el
at i
onshi p wi t ht he nat i
onalgov ernmentbei ng f undament all
y
differentf rom t hatoft heARMM.I ndeed, BJEi sast atei nal lbutnameasi tmeet sthecr iteriaofa
statel ai ddowni ntheMont ev ideoConv ent i
on, namel y,aper manentpopul ation, adef inedt err i
tory,
agov er nment , andacapaci tyt oent eri ntor elationswi thot herst ates.
Evenassumi ngar guendot hatt heMOA- AD woul dnotnecessar ilysev eranypor ti
onof
Phi l
ippi net er ri
tory,t hespi ritani mat ingi t-whi chhasbet r
ay edi tsel fbyi tsuseoft heconceptof
associ ation-r unscount ert ot henat i
onal sov er eignt yandt erri
torial integrit
yoft heRepubl ic.
TheMOA- AD,mor eov er,woul dnotcompl ywi thAr t
icl
eX,Sect ion20oft heConst itution
sincet hatpr ov i
siondef i
nest hepower sofaut onomousr egionsasf ollows:
SECTI ON 20.Wi thini tst err i
torialj ur isdict i
on and subj ectt ot he pr ovisions oft his
Const i
tutionandnat ionall aws,t heor gani cactofaut onomousr egi onsshal lprov idef or
l
egi slativepower sov er :
xxx
9.Suchot hermat tersasmaybeaut hor izedbyl awf ort hepr omot i
onoft hegener al wel fare
oft hepeopl eoft her egion.( Under scor ingsuppl ied)

Againont hepr emi set hattheBJEmayber egar dedasanaut onomousr egi


on,theMOA- AD
wouldr equireanamendmentt hatwoul dexpandt heabov e-
quot edpr ovi
sion.Themer epassageof
newl egislat
ionpur suantt osub- paragr aphNo.9ofsai dconst i
tutionalprovisi
onwoul dnotsuf f
ice,
sinceanynewl awt hatmi ghtv estint heBJEt hepower sfoundi nt heMOA- ADmust ,itself
,compl y
withot herprovisionsoft heConst itution.Itwoul dnotdo,f orinstance,t omer elypassl egislati
on
vesti
ngt heBJEwi thtreaty-maki ngpoweri nordertoaccommodat epar agraph4oft hest r
andon
RESOURCESwhi chst ates:" TheBJEi sf r
eet oent erintoanyeconomi ccooper ati
onandt rade
relat
ionswi t
hf oreigncount ri
es:prov i
ded,howev er,thatsuchr elationshipsandunder st andingsdo
notincludeaggr essi onagai nsttheGov ernmentoft heRepubl icoft hePhili
ppinesxxx. "Underour
constit
ut i
onalsyst em,itisonl ythePr esi dentwhohast hatpower .

ATTY.VI
LLEGAS 42
I
NTERNATI
ONALENVI
RONMENTALREGULATI
ONS
ANDNATURALRESOURCESDI
GESTS

BANGUSFRYFI
SHERFOLKETAL.
,VS.JUDGELANZANASETAL.
G.R.No.131442      
July10,2003
Ponente:Car
pio

FACTS:DENRRegi onalExecut iveDi r


ect orPr i
ncipei ssuedanEnv i
ronment alCl earanceCer ti
fi
cate
(ECC)i nf avorofNapocor ,author izingt hecor porationt oconst ructat empor arymoor ingfaci l
i
tyin
theMi noloCav ei nPuer toGal er a.TheSangguni angBay anofPuer toGal erahasdecl aredt he
Mi noloCav ewhi chwasamangr ov ear eaandbr eedi nggr oundf orbangusf ryaeco- touri
stzone.
Themoor ingf acil
it
ywoul dser veast het empor arydocki ngsi teofNapocor 'spowerbar ge.The
Bangus Fr y Fisher folk,cl aimi ng t o be f i
sherfolks f rom Mi nolo,Puer to Gal era,sought
reconsi derati
onoft heECCi ssuance,whi chwasdeni edbyt heDENR.TheBangusFr yFisher folk
thenf il
edacompl aintwi t
ht heRTCofMani l
af orthecancel l
at i
onoft heECC, forthei ssuanceofa
wr i
tofi njuncti
ont ost opt heconst r
uct i
onoft hemoor ingf acil
ity,andf ur therpr ayedf ort he
demol i
tion ofmoor ing st ructur es t hathas al ready been bui lt
.By t hi st ime,t he pr ov i
ncial
gov ernmentofMi ndor o mani fest ed thati twast heoneunder taking t heconst ructi
on oft he
moor i
ngf acil
ity.
ThePr ovincialGov ernmentofMi ndoromov edt odi smi sst hecompl aintf ort hefailureto
exhaustadmi ni
strativer emedi es, r enderingt hecompl aintwi thoutcauseofact ion,andt hatMani l
a
RTChadnoj uri
sdictionast hemoor i
ngf acili
tyisout sidei tster r
itori
al j
ur i
sdiction. The RTC
dismi ssedt hecompl aintf orf ailur etoexhaustadmi nist
rat i
v er emedi essi ncet herewasnoappeal
bef oret heDENRSecr etarypr iort of il
ingt hecasewi tht het ri
alcour t.TheBangusFr yFisher folk
cont endt hattheyar eexemptf rom f i
li
nganappealwi tht heDENRSecr etarybecauset heissuance
oft heECCwasapat entv iolationofexi sti
ngl awsandr egul ations,oneofwhi chwasSect ions26
and27oft heLocal Gov ernmentCode.

I
SSUE/S:
1. W/NRTCer redindismi
ssi
ngthecompl
aintf
orlackofcauseacti
onandlackofj
uri
sdict
ion.
–NO.
2. Whethert
hesangguniangbay
anofPuert
oGal er
awasr equi
redtoappr
ov et
heconst
ructi
on
ofamoor i
ngf aci
l
ity–NO,Secti
ons26and27ar einappli
cabl
etoproject
swhicharenot
env
ironmental
lycri
ti
cal

RATI
O:
Jur i
sdicti
onoft heManil
aRTCov ert
heCase
TheMani l
aRTChasjuri
sdi ctiont odet ermi net
hev al
idi
tyoftheissuanceoftheECCbecauset he
offi
ceoftheRegi
onalDirect orPr i
ncipei sinMani l
a,wit
hinitster
ri
tori
alj
uri
sdicti
on.However
,the
j
urisdi
cti
onofRegi
onalTrialCour t
st oi ssueinj
unctiv
ewr i
tsisli
mitedtoactscommi tt
edorabout
tobecommi t
tedwit
hinthei rjudicialregion.Theref
ore,i
tcouldnotissueaninjuncti
vewri
tagai
nst
theDENRorNAPOCOR.

Exhaust
ionofAdmi ni
str
ativeRemedies
UndertheDENRPr ocedur
alManual,afinaldeci
si
onofar egi
onalDi
rect
ormaybeappealedtothe
Off
iceoftheSecret
aryofDENR.Thef ai
lureofthi
smet hodofappealdepri
vedtheDENRSecret
ary
torevi
ewthedecisi
onofhissubordi
nate.Thisomissi
onr ender
sthecomplaintdi
smi
ssi
bleforl
ack
ofcauseofact
ion.

OntheAllegedPatentI l
legal it
yoft heECC
Whi l
ethepat entil
legali
tyofanactexempt sapar t
yf rom compl yingwithther uleonexhaust i
onof
admi ni
strati
veremedi es,thi
sdoesnotappl yinthepr esentcase.Congr essi ntr oducedSections26
and27i nt heLocalGov ernmentCodet oemphasi zet hel egislat
iveconcern“ fort hemaintenanceof
asoundecol ogyandcl eanenv i
ronment.”Thesepr ov i
sionsr equireeverynat i
onalgovernment
agencyorgov ernment -
ownedandcont rol
l
edcor por ationt ohol dpriorconsul tationswit
ht helocal
governmentuni tconcer nedandt osecurethepriorappr oval ofi
tssanggunianbef orei
mplement i
ng
“anypr oj
ectorpr ogr am thatmaycausepol l
ution,cl imat i
cchange,depl etionofnon- renewable
resources,lossofcr opland,rangel
and,orf or
estcov erandext incti
onofani malorpl antspecies.”

ATTY.VI
LLEGAS 43
I
NTERNATI
ONALENVI
RONMENTALREGULATI
ONS
ANDNATURALRESOURCESDI
GESTS

Again,Sect i
ons26and27donotappl yt ot hi
scasebecauseaspet i
ti
onersadmi t
,themoor i
ng
faci
li
tyitselfi
snotenv i
ronmental
lycrit
icalandhencedoesnotbel ongt oanyoft hesixtypesof
proj
ectsment i
onedint helaw.Thereisnost atutor
yr equir
ementf ort heconcer nedsanggunianto
approvet heconstructi
onoft hemoor i
ngf acili
ty.Itisanothermat teriftheoper ati
onoft hepower
bargeisati ssue.Asanenv i
ronmentall
ycr it
icalprojectthatcausespol luti
on,theoper ati
onofthe
powerbar geneedst hepri
orapprovaloft heconcer nedsangguni an.Howev er
,whati sbeforethi
s
Courtisonl ytheconstructi
onofthemoor ingf aci
li
ty,nottheoperationoft hepowerbar ge. Thus,
theissuanceoft heECCdoesnotv i
olateSect ions26and27ofRANo.7160.

ATTY.VI
LLEGAS 44

You might also like